Download as pdf or txt
Download as pdf or txt
You are on page 1of 74

‫تجميعات قلوري تيم لشهر ‪ 11 – 10 – 9‬من سنة ‪2019‬‬

Thyroid
1-After total thyroidectomy develops hypocalcemia despite calcium replacement more than once .
What to check next?
Measure serum magnesium level .

( Persistent symptoms if only calcium deficiency corrected . When both calcium and magnesium
are administered , hypocalcemia signs improve )

2 - Patient admitted for thyroid surgery because of progressive compression symptoms .


Hemithyrodectomy done and biopsy showed 8mm papillary carcinoma , whats the next step ?
Follow up
Key word:
(papillary carcinoma → after hemothyriodectomy no need for radiation )

3- After right hemithyroidectomy On biopsy found follicular lesion 8cm from the?
Complete thyroidectomy.

4-Pt for thyroidectomy due to cancer and with mitral valve prolapse . Prophylaxis against infective
endocarditits …..?
a. No need b.Ampicillin c. Ceftriaxone

5- Pt after thyroidectomy , develops hypocalcemia despite calcium replacement more than one time
, what to do next ?
Check serum magnesium level

6- Pt came after thyroid surgery with dysphagia and horseness of voice. Which nerve is injured?
Recurrent laryngeal nerve .

7- A patient post thyroidectomy can not make high pitch sounds. Damage to which nerve is
responsible?
Superior laryngeal nerve.

(external branch of SLN is nerve most commonly, 0- 25 % , injured in thyroid surgery resulting in
inability to lengthen a vocal fold → inability to create high-pitched sound .
Speech therapy is the only treatment )

8- Patient with hyperthyroidism for 10 m and treated with anti hyperthyrdism But his condition did
not improve ultrasound done and show multi nodule and diagnosed as Graves’ disease what is the
best next step ?
A.total thyroidectomy B.subtotal thyroidectomy C. iodine therapy ✅ D.increase the dose of
drug
radioactive iodine ‫ مانفع نروح لالشعة‬+ ‫ امشوا بالترتيب اول حاجة ادوية تنزل مستويات الغدة‬hyperthyodism ‫خطوات عالج‬
‫مانفع نعمل استئصال جزئى للغدة‬

2019 ‫ من سنة‬11 – 10 – 9 ‫تجميعات قلوري تيم لشهر‬


Treatment Graves’ disease :
First start with propylthiouracil (PTU) or methimazole (MMI)
PTU also inhibits peripheral deiodination of T4 to T3
• symptomatic treatment with B-blockers
• thyroid ablation with radioactive 131I if PTU or MMI trial does not produce disease
remission
• subtotal or total thyroidectomy (indicated rarely for large goitres, suspicious nodule
for CA, if patient is intolerant to thionamides and refusing RAI ablation)

9-Neck mass biopsied showing (thyroid folicullar cells)? Ectopic thyroid ✅

(if question states that there is lymph node involvement with follicular cells → metastases )

10-Thyroid nodule hard ultrasound report Next step? A.FNA ✅ B.lobectomy


Key word: hard or cold or solider= FNA
Thyroid Nodule

Thyroid Function Tests

Normal Hyper -functioning

FNA Thyroid Scan

Cold Hot = Hyper-functioning toxic

Start Antithyroid therapy / Surgical Excision

11-Pt with symptoms of hyperthyroidism and he is on medical treatment for 10 months with no
improvement what next? Radioactive iodine✅
‫لو الحالة وحدة حامل ال نختاره الن ممنوع على الحوامل‬

12-Thyroid enlargement (hot nodule) FNA normal What is next step? start antithyroid therapy

13-Thyroid nodule, cold, investigation? A. FNA

14-Patient with thyroid nodule asymptomatic And thyroid function test all within normal
US done . What’s next?
A. FNA.(bcause thyroid function normal)✅ B. Start antithyroid medication. C. thyroid scan
FNA is recommended for all palpable solitary or dominant nodules, independent of size. FNA is
preferred over thyroid scan or ultrasonography as the initial diagnostic test for thyroid nodules. ...
When TSH is suppressed or the patient is thyrotoxic, a nuclear scan maybe indicated before FNA
2019 ‫ من سنة‬11 – 10 – 9 ‫تجميعات قلوري تيم لشهر‬
15-Pt with thyroid nodule and hyperthyroidism lab , radioactive iodine shows the nodule hot, but all
the remaining thyroid is cold. Initial Management: antihyperthyroid drugs in Hashimoto’s increases
risk of?
Primary thyroid lymphoma

16-A patient post total thyroidectomy developed neck swelling 5 hours after the surgery. most
appropriate?
bedside evacuation
key word: hematoma
17- Medullary thyroid cancer mangement? Total thyroidectomy

18- Normal follicular thyroid cells, how to manage? Hemithyroidectomy

( if benign -→ follow –up , if malignant → radioactive iodine )

19- Female , with lateral mass (lymph node) , thyroid was normal , biopsy showed normal
follicular cells, what is the diagnosis? metastasis

20-Normal thyroid, cervical lymph node bx showed normal follicular thyroid cell dx
on specimen it’s shows follicular thyroid cells? metastases✅

21- Pt with normal thyroid and swelling in the neck this swelling is cervical LN and FNA showed
normal follicular thyroid tissue?
A- Remove this LN B-Refer to surgery✅ C- Radio D-Chemo

22-Thyroid nodule measuring 4 cm? hemithyroidectomy

23- Thyroid nodule measuring less than 1 cm? follow up

24- All things are normal but have pain in his neck . They check his left neck and found small mass
5*7 mm what is the most important thing to do ?
A. FNA. B. complete his thyroid investigation✅

25-medullary thyroid carcinoma management? total thyroidectomy

26- Hematoma after thyroidectomy. Next? bedside evacuation✅

27- Patient post thyroidectomy, during the ward he has expanding neck swelling with difficulty of
breathing what to do? Immediate bed side opening of wound
Key word: hematoma

28-High TSH , low T4 ,T3 , high ESR diagnosis? Subacute thyroiditis

29-Complication after thyroid surgery , Hoarseness of voice ? recurrent LN

30-Insufficient thyroid FNA, what would you do next? repeat FNA

31-Strongest indication of thyroid surgery ?


2019 ‫ من سنة‬11 – 10 – 9 ‫تجميعات قلوري تيم لشهر‬
A. -pediatric B. -presence of eye symptoms. C. -presence of anti tsh . D. -failed
antithyroid meds.✅ A is correct

32- pt with hyperthyroidism feature and irregular pulse, what to first test? Thyroid function tests
Key word : signs of hyperthyroidism and irregular puls

33- 25 years female with thyroid nodule TSH and T4 normal , FNA cytology done and
according to Bethesda classification ((Stage IV )))what is the most accurate management?
LOBECTOMY lobe= 4 = iv

34- Bethesda stage VI ?


NEAR TOTAL THYROIDECTOMY

❖ Stage lV =LOBECTOMY (‫ حرف‬L ‫ الي هو اول حرف مع أول‬I ‫ بعدين يجي‬V)


❖ Stage III=repeat FNA and observation (‫ كلمة‬repeat ‫ حرف‬I ‫) متكرر ثالث مرات‬
❖ Stage VI= Near total thyroidectomy ( ‫ نشيل الغدة كلها‬، ‫) المرحلة االخيرة‬

35- Unilateral neck swelling in the RT side by investigations : hot thyroid nodule TSH is high, T3,
T4 low No LN enlargement Treatment?
A- antithyroid drug✅ B- RT thyroidectomy C- Hemithyroidectomy D- radioactive iodine

36- Patient came for 3 months neck pain, thyroid function test all normal, ultrasound:
single solid thyroid mass in right lobe. Best next management ? A.Thyroid scan. B.FNA✅.

37- Euthyroid thyroid nodule, what you will do? Fine Needle Aspiration ✅
‫ ربط‬: Euthyroid ‫ بما انه طبيعي نحتاج الى ابرة رفيييعة جدا‬، ‫ يعني وظائف الغدة طبيعية‬Fine needle

38- pt with enlarged lymph node and normal thyroid FNA of LN showed norm'l follicul'r thyroid
tissue ( metastasis) :
A-Abx. B- Remove the LN C- Refer to surgery ✅

39- toxic nodule and rest of the gland is suppressed ?


If small go for radio. If large or causing compression, go for hemithyroidectomy.

40-Patient with congested throat symptoms and 2 cm palpable cervical lymph nodes. What is the
most appropriate investigation?
FNA of the lymph nodes✅

41-Pt with diffuse thyroid enlargement on exam there is 1 nodule in each lobe, labs showed
hyperthyroidism what will you do?
A. Thyroid scan. ✅ B. FNA from both nodule

42- 48 yo lady with diffuse goiter, high T4 low TSH, US show bilateral thyroid nodules , right 3x4
in size , left is 1x2 size what to do?
A. FNA both B. FNA the larger one C. total thyroidectomy✅
Key word: big saize

2019 ‫ من سنة‬11 – 10 – 9 ‫تجميعات قلوري تيم لشهر‬


43-Patient with spiral fracture, parathyroid hormone high and Ca high.. Most likely diagnosis?
Parathyroid adenoma ✅
Key word: parathyroid hormones and Ca high

44-parathyroidectomy pt have increase in Ca ?


Missed adenoma

Key word : post Para thyroidectomy+ increase Ca

45-Indication of elective parathyroid gland removal?


Evidence of osteoporosis.

46-After near total thyroidectomy the patient have persistent calcium decreased despite giving him
calcium multiple times. What to give him ? Mg

❖ Post operative complications:


47- Pt Postoperative blood transfusion develops fever and pain at site of infusion ?
febrile non hemolytic reaction

48-Pt post op triple A repair become unstable even with fluid replacement and have negligible urine
in cath decreased vascular resistance and increased cardiac out put.Type of shock ?
septic✅

49-female had a hip fracture now had DVT as I remember now she is on heparin but her platelet is
low 58 so what will you do?
A. Shift to SC enoxaparin B. Stop heparin and start other anticoagulant✅
Key word: heparin induce thrombocytopenia

50-Patient morbid obesity for treatment what will u do before?


endoscope ✅

51-Pt with progressive abdominal pain post sleeve, what to do?


CT .

52- pt. with heart failure and admitted for surgery for some disease and connected
to IV fluid, post-op 2 days later complained of SOB and bilateral basal.crepitation,
how could this be prevented?
A. IV Furosemide immediate post-op B. monitoring IV fluids daily✅
‫مرض مثل فشل القلب والكلى انتبهوا يشربوا او ياخذوا سوائل كثير الن فشل االعضاء هذه صعب جدا تطلع الماء برة الجسم فيتراكم‬
‫فراقبوا كمية السوائل‬،‫بالجسمم‬
53-Scar post surgery increasing in size?
Keloid

2019 ‫ من سنة‬11 – 10 – 9 ‫تجميعات قلوري تيم لشهر‬


54-during laparoscopy doctor just start the procedure pt become hypotensive 84/50 what’s the
cause?
A. cold gas B. increase preload C. rapid inflation of the abdomen✅
Key word: Peritoneal stretching > vagal stimulation

55-Most common complication post operative?


A-wounds infection. B- pneumonia'. C-Atelectasis ✅

56-24 hrs postop patient has dyspnea and hypoxia cause? lower lobe atelectasis ✅

57-patient with abdominal mass after lifting heavy objects mass not change with cough. Dx?
a) rectus sheath hematoma✅ b) hernia
- key word: not change with cough
- 58-Pt 56 c/ o sudden RLQ pain after lifting heavy object ; and mass in RLQ ; Cough
negative ; abd muscle tense even with clenching . He is on anticoagulation due toA fib
What the most appropriate mgx ?
(Rectus sheath hematoma treatment = rest and analgesic and stop anticoagulants

59- Morbid obese pt with GERD wich procedure is good for him ?
Roux en y

60-13yo obese boy found to have large hiatal hernia and grade 3 reflux asking
about bariatric surgery choice: A. Roux-en-Y Gastric Bypass

61-Obese male tried lifestyle not effective He is diabetic hypertensive what to do , Weight 125 ,
Hight 173 , What next ? ( You should calculate BMI )
A. Medication lower weight B. bariatric surgery ✅
2
( BMI = Weight ( kg ) ÷ Height squared ( m ) )

BMI Category

< 18 Underweight
18 to < 25 Healthy weight
25 < 30 Overweight
30 to < 35 Stage I obesity
35 to <40 Stage II obesity
=40 Stage III obesity ( formerly “
morbid obesity “ )

62-Women obese HTN diabetes controlled BMI 28 Tried excersise and lifestyle
modification but didn’t work What to do?
A. Bariatric surgery B. Orlistat (drug for obesity)✅

63-Case of pt with constipation for 1 week then developed watery diarrhea, he has tender abdomen
and small distention ?
A. Constipation ✅ B. Acute colitis
2019 ‫ من سنة‬11 – 10 – 9 ‫تجميعات قلوري تيم لشهر‬
64-29 Years old male did sleeve gastrectomy 6 days ago, came to the ER complaining of persistent
vomiting since operation, on exam nothing significant,ABG normal( there were numbers),Us
normal. Best management?
Reassure
‫ ف هنا االعراض طبيعي ف نطمن‬.‫الن طبيعي ا مع عمليات تكميم المعدة يح صل هذه االعراض وهو استمرار القيء وتعب وهذيان‬
‫ المريض بينما لو حصل له الم شديد جدا في بطنه او استفرغ دم هذا ماينفع نطمنه الزم يروح للمستشفى‬.

65-Diabetic and HTN .., BMI > 40 (you calculate .. height and weight were given) .. he tried to
reduce weigh but still obese .. he is not on medications .., NEXT ?
Sleeve surgery ✅✅
‫ ويشكتي من اعراض السمنة‬35 ‫ او اذا شخص كتلة جسمه فوق‬، ٤٠ ‫شوفوا متى نعمل عملية السمنة ؟؟ اذا شخص كتلة جسمه فوق‬
‫ نختار‬٣٥ ‫ بينما لو اقل من‬، ‫ ومضاعفاتها مثل سكر وضغط وكليسترول‬A

66-Patient done gastric sleeve after 3 days come irritable and have some bloating what to do?
reassurance ✅

67-Patient morbid obesity for treatment what will u do before?


endoscope ✅

68-70 year old pt fall on the ground surgeon will do hip replacement surgery before surgery
procedure what is the best prophylaxis thrombolytic drug for him ?
LMWH , enoxaparin .
(IF patient with CKD -→ UFH )

69-After surgery by 4 days patient developed MI .. what to give ?


angioplasty

70-Pt with pelvic fx and bleeding per rectum >> urethrogram > retroperitoneal urethra injury Mx?
A-Folly's cath B-Supra pubic cystostomy✔ C-Labroscopic repair

( make sure its cystostomy not cystectomy )

71-Membranous urethra injury management?


A. Folly Catheter B. Retropubic catheter ✅ C. Retropubic repair

72- Membranous urethra injury is? posterior injury

Injury to male urethra is either


Injury to Posterior Urethra Injury to Anterior Urethra
( at / above membranous urethra ) (penile / bulbar)
Almost associated with pelvic fractures Straddle injuries where urethra crushed
(1.5- 10 % of fractures) between pubic bones and fixed object
Concomitant bladder injury in 15% Maybe blunt/ penetrating

2019 ‫ من سنة‬11 – 10 – 9 ‫تجميعات قلوري تيم لشهر‬


73- Old patient had percutaneous cholecystostomy, presented after 24hrs with upper GI bleeding
what is the most important investigation? A. angiography (correct)

74-Female with thyroid mass asymptomatic us shows hard nodule what are the next steps?
A. Thyroid scan with iron B. FNA ✅

❖ Operative
75-Indication of surgery for Aortic stenosis? A. pt symptoms✔(correct) B EF less than 50

76-after surgery what is best dvt prophylaxis? enoxaparin and mechanical

77-lady was admitted and discharged just a day before she experienced severe right leg pain and
edema. (Basically she was discharged and the next day she got this presentation) diagnosis?
A. DVT✅ B. Acute thrombotic disease C. Acute embolic disease

78-Patient has resection rectal surgery , next day has left leg pain what do you give ?
Enoxaparin

79-Patient was stabbed in the right lower side of his chest, FAST revealed free abdominal fluid ?
A-Thoracotomy B-Laparotomy ✅✅.

STAB WOUND TO ABDOMEN

Patient Stable Patient Unstable

CT FAST

If free abdominal fluid

LAPAROTOMY

80-pt complain of melena and epigastric pain diagnosed as peptic ulcer and received ppi but not
improved endoscpy done showed multiple ulcers in the antrum what is the most accurate
management : a.antrectomy✅
‫ في‬ulcer ‫ فالسؤوال هنا يقول انه حصل‬antrum ‫ ؟ المعدة تقسم الى ثالث اجزاء اخر جزء من المعدة اسمه‬antrum ‫تعرفون ايش هو ال‬
.antrectomy. ‫ عالجها‬antrum ‫ فالعالج القرحة هذه ه باستئصال الحزء الي حصل فيه قرحة ومن اسمها‬antrum

81-Patient known case of peptic ulcer disease in medications but failed to respond, upper endoscopy
done and revealed multiple ulcers in antrum. What is the best treatment?
A. Pyloroplasty and vagotomy✅✅✅ B. Total gastrectomy C. partial gastrectomy
. ‫ وهللا ااعلم‬A . ‫ اختاروا‬antrectomy ‫لو مافي‬
Vagotomy was a way to reduce the acidity of the stomach, by denervating the parietal cells that produce
acid. This was done with the hope that it would treat or prevent peptic ulcers. It also had the effect of
reducing or eliminating symptoms of gastroesophageal reflux in those who suffered from it.

82-anterior abdominal stab wound , the omentum is bulging out through wound ?
Exploratory laparotomy✅

2019 ‫ من سنة‬11 – 10 – 9 ‫تجميعات قلوري تيم لشهر‬


83-Pt diabetes pregnant and baby has diaphragmatic hernia and baby driver and do for him
resuscitations and stabilization what next step ? surgery

- 84-Pt is with non-acth dependent cushing for rr adrenalectomy ..what is the postoperative
management?
A. postoperative fludrocortisone B. postoperative metatone C. preoperative hydrocortisone✅
D. pre operative (drug for pheochromocytoma forget its name )

- 85-Echinococcus syptomatic ((calcified)? Surgery(cystopericystectomy)✅

- 86-Case of liver cyst (echinoccosis) treatment ? Albendazole ✅


(‫= اذا )هاي مع قلب‬. Hydatid ‫ من اسم‬+ Hy= ‫ قلب‬، ‫ = ألب‬Albendazole = ‫نربطها‬

- 87-hydatid cyst 10*15 cm with multiple ((daughter cyst )) inside the main one. What is the
treatment?
deroofing surgery

- 88-Picture of Ct hydatid liver cyst: One large, manage:


A. open cyst deroofing ✅ B. albendazole
‫ لكن لو كبير الزم اشيله كلمة‬، ‫ وهو يختفي باذن هللا‬alben ‫ صغيرة نعطيه دواء‬cyst ‫لو كان‬
.‫ المتجمع داخلها‬pus ‫ يعني افتح الكيس بالمنظار واشيل‬deroof

- 89-Pic of Hydatid cyst on U/S it was big but daughter cysts was mentioned. What should you do:
Surgical Deroofing
= daughter. ‫ديرفينج = ظفييرة‬

- 90-amebic liver disease, what drug to give ‫؟‬Metronidazole


‫ و متروا حصل فيه حريق نطفي المتروا ب الميبوا‬، ‫ ميبوا كريم حروق‬، ‫ربط‬

- 91-Classic amebic abscess scenario with clinical and imaging findings. Management?
2019 ‫ من سنة‬11 – 10 – 9 ‫تجميعات قلوري تيم لشهر‬
○ Metronidazole ✅. ○ Drainage. ○ Aspiration

- 92-Long scenario with liver abscess culture reveals anti amoeba histolytica treatment?
metronidazole ✅✅

- 93-Large cyst >10cm or complicated? Surgery

- 94-Patient with hydatid cyst, complex with septations and 10x15 in size, initial step in treatment?
A. Cephalosporin B. Mebendazol ✅✅ its type of albenazol
C. Percutaneous aspiration D. Percutaneous drainage

❖ Peripheral artery disease:


- 95-Acute limb ischemia with absent distal pulse, diminished sensory and altered motor ex, mx?
A. Embolectomy. B. Above knee amputation. C. Cath and thrombolysis D_heparin ✅
Key word: acute limb ischemic = heparin
- 96-pain ,paresthesia ,pallor ,,Hx of MI before weeks ,Dx?
A. Acute arterial thrombosis✅ B. Acute arterial embolism C. DVT
Key word: 3ps= pain, paresthesia , pallor =acut ar throm

- 97-An elderly man came with lateral malleolus ulcer, he is hypertensive but
not diabetic What is the first thing to do:
A. venous duplex B. check pulse ✅ C. ABI

- 98-Sudden right lower limb pain. Diminished sensation + altered motor. Pt on anticoagulant for
afib.Best next step: A. Ct angio✅ B. Us C. emblectomy
Key word: Afib = CT angio
- 99-What is best to confirm arterial disease? Ct angio✅

- 100-Patient with HTN and diabetes came with ingrown toe : what is the most important thing to do
before the surgery : A- check pulse ✅ B- check the other
toe
Key word: ingrown= check pulses

- 101-Old male with HTNand DMpresented with claudication examination reveals


right femoral intact pulse and diminished popliteal and distal pulse and left diminished
pulse intervention?: A. CT angio ✅ B. conventional angio C. mra D. venous US

- 102-50 years old patient, know case of DM, HTN presented with leg pain associated
with hair loss , ABI more than 0.9, what is your diagnosis:
2019 ‫ من سنة‬11 – 10 – 9 ‫تجميعات قلوري تيم لشهر‬
A. acute thrombus ischemia B. Chronic limb ischemia ✅
Key word: ABI (ankle – brachial pressure index) more than 0.9 =cronic
Normal ABI= 0

- 103-Diabetic patient with pseudo hyper epithelialization in situ? ulcer Debridement

- 104-lateral malus ulcer with abaent pulse most IMP investigation? A. CTA✅
Latral mean ischmic ulcer
- 105-Dm pt with unilateral leg erythema Increases in dependant position, Cold, Femoral pulse is
present, distal pulse can be palpated No tenderness, no swelling, no fever Otherwise unremarkable
Dx?
A.Cellulitis B. Arterial insufficiency C.Superficial thrombophlebitis✅

Key word: erythema , cold pulse present


no tenderness no swelling no fever

- 106-Patient with diabetes and hypertension smoking 40 c per day came to er complaining of leg
pain on examination on lower limp the pulse was intact on femoral and pupletial artery’s and
diminished on distal pulse first question was what is the best appropriate next step ?
A. morphin B. heparin ✅ C.ct angio D.vascular us

- 107-Pt have deep thigh injury what to do?


A.torniquete B. put pressure above site of injury ✅ C.put pressure on injury
Key word: to stop bleeding
- 108-Pulse abdomen mass in middle abdomen what (next step to confirm) management ?
A. angiography B. CTangio ✅✅ C. US
Key word: dx is abdominal aortic aneurysm intial US,
confirmation and management CT angio

- 109-Pulsatile abdominal mass Abd X ray showed no air level What (((nexst step ))) investigation to
order:
A. abd Us✅ B. CT angio

- 110-60 years old patient come with epigastric pain radiating to back . He’s smoker and long
standing for DM and HTN . Abdominal examination show pulsatile subra umblical mass . What’s
Dx ?
A. Abdominal aortic aneurysm✅ B. secondary liver metastasis C. renal cell carcinoma

- 111-Surgery was done for diabetic septic foot ,no wound healing for 2month what is the cause?
A.Neuropathy B. Poor blood supply ✅✅

- 112-PT admitted due to MI after 2 days of discharge developed sever pain in his leg Dx?
1/ acute arterial thrombosis 2/ Acute Arterial Embolus ✅ 3/ DVT

2019 ‫ من سنة‬11 – 10 – 9 ‫تجميعات قلوري تيم لشهر‬


Areythmia ‫ الي تعمل‬Mi ‫ بسبب مضاعفات‬Embolism ‫ فغالبا جلطة تحركت من القلب‬MI ‫هنا قال انه بعد يومين من‬
Most commen couse of embolism from heart
Thrombolism ‫ يعني من زمان فيمكن يمشي اكثر مع‬MI ‫الن فيه سؤوال مشابه وقال عنده هستوري ب‬
And also history of Dm and HTN

- 113-claudication with distance peripheral artery disease. ABI? ankle brachio index

- 114-what is the first symptom to be find in compartment syndrome?


a. pain ✅. b. paresthesia between toes. c. swelling

intac puls or absent puls:‫ لما يقولك السيناريوا‬doplex ‫ و‬C.t angio ‫متى نختار‬
absent puls‫لو لقيت بالسيناريوا يقول‬
‫(( على طول اعمل له‬next step )) ‫ بعدها‬puls ‫يعني فحصته بالعيادة لقيت مافيه‬
doppler ultrasound
‫الن الدوبالر اسرع واسهل‬
‫اذا عملت له دوبالر وماوضح معايا اي شريان بالضبط المسدود اروح للمرحلة الي بعدها واسوي‬
C.t angio
.‫ يوضح لي فيه انسداد وضيق لكن مايوضح اي شريان بالضبط‬doppler ‫الن‬
‫ نختار‬investigantion ‫لذلك لو سال ايش افضل‬
C.T angio
US Doppler .‫ طبعا‬next step ‫لو قال‬
lymphatic‫ او‬venus ‫ معناها الشريان سليم ولكن المشكلة في‬puls intac ‫بالنسبة لو ال‬
‫وخاصة لو قال‬
non poitting Edema , thikness , pigmentantion skin =
mean there is lymohatic obstruction
lymphatic‫هذه الكلمات تمشي مع‬
‫ الي اطلبها اول اول شيء‬investignation ‫ايش‬
Ultrasound or venus ultrasound
DVT‫عشان استبعد‬

- 115-Crush injury to his feet , after stebalization of the patient , o/e cold and decreased pulse in the
affected foot . What is the (((best next step )) u will do ?
A. ct angio B. angio C. duplex US ✅ D. compartment compression
Key word: if there swelling and pain =D
If hard signs = A

- 116-with Reversible Eye manifestation for 20 min?


TIA ✅ ) transient ischemic attack (
‫ يعني عابر شيء مؤقت ويرجع للطبيعي‬transient ‫من اسمها‬
- 117-Female patient after cryotherapy for varicose 2years ago complainig of leg and foot pain what
nerve is damaged ? A- siatic. B- saphenus ✅. C- femoral D- obbirator
= sphen ‫ ازرق = سفينة‬varicose vien ‫لون‬
- 118-Female 30s asking for cosmetic treatment of varicosity in her thigh, asymptomatic, no other
varicosity, how would u investigate?
A. Duplex ✅ B. CT venography C. No need for further investigation

2019 ‫ من سنة‬11 – 10 – 9 ‫تجميعات قلوري تيم لشهر‬


- 119-lady who have Spider nevi she wants to treat it cosmetically:( they mean before treatment what
should you do ) ? A) No need for investigation B) US doppler✅

- most common location for arterial ulcer >> Lateral. No pulse>> arterial
- Most common location for venous ulcer >> medial. pulse >>> Venous
*But the pulse will be the clue .*
- arterial ulcer= Lateral + No pulse
- venous ulcer = medial + pulse
‫= يعني جدا حساس على الحافة يادوبك لو سار له اصابة راح يهدد حياة الشخص‬latral ‫ربط= الشريان على الحافة‬
medial‫ عادي في االمان‬venus ‫بينما‬

-
- 120-female present with bilateral lower limb swelling non putting not known to have any medical
illness :
A-CT angio B-duplex ultrasound✅
Key word:Non pitting edema = duplex
Treat non pitting edema= if no swelling =bandage if swelling =lymphatic message
Best investigation=ct
- 121-Patient with unilateral left leg non-pitting edema, skin thickening and hyperpigmentation
(dermal fibrosis) for 2 months. Next best appropriate step?
○ CT angiography. ○ MRV. ○Duplex US ✅ (initial and the next)
Key word: Non-Pitting edema is due to Lymphedema, Ultrasound is useful to exclude other etiologies
like DVT, venous insufficiency and can also help in identifying tissue changes\ and masses that might
be the cause of lymphatic compression.

- 122-Patient with unilateral left leg non-pitting edema, skin thickening and hyperpigmentation
(dermal fibrosis) for 2 months. Management?
A- Lymphatic bypass B- Compression bandages (combination of physical therapies; two-stage
approach) ✅
NOTES: Nonspecific treatment of lymphedema includes exercise, elevation, compressive garments,
manual lymphatic drainage, intermittent pneumatic compression, and surgery (excisional
procedures, microsurgery).
2019 ‫ من سنة‬11 – 10 – 9 ‫تجميعات قلوري تيم لشهر‬
- 123-same question but swelling unilateral ask about management?
A- lymph by pass. B- anticoagulant. C-lymphatic massage ✅

Treatment focuses on reducing the swelling and controlling the pain. Lymphedema treatments include:
1) _ Exercises. Light exercises in which you move your affected limb may encourage lymph
fluid drainage
2)_ Compression garments. Long sleeves or stockings made to compress your arm or leg encourage
the flow of the lymph fluid out of your affected limb.
‫ الي هو (شراب ضاغط ) مثل شراب طويل يكون مرة ضيق على الرجل‬.
3)_ Massage. A special massage technique called manual lymph drainage may encourage the flow
of lymph fluid out of your arm or leg.

- 124-Female on her fifties develop unilateral leg non-pitting edema with skin discoloration
(dark) and thickening of skin ,mx?
-lymphathic bypass. -anticoagulant -lymphatic massage and pressure dressing ✅✅

- 125-presentation lymphedema, what is the important thing in hx to ask about?


A. Medication B. Family hx C.Trauma ✅✅ D. Smoking
: ‫ مثل‬truma events ‫ للمرة االولى بعد ما يحصل اي‬lymphedema ‫ممكن يحصل‬
such as bruises, cuts, sunburn, and sports injuries
‫فمهم في الهيستوري نسالهم هل حصل سابقا اي تروما ؟‬
- 126-Bilateral great saphenous vein mange ?
A. sclerotherapy B. something endoscopic C. ablation ✅
Key word:Any bilateral great saphenous = ablation
Cosmetic =sclerotherapy
- 127-Lateral malleolus ulcer. Diabetic. Intact pulse. Best initial test?
A- Douplex affected leg✅ B- Douplex both legs C- CT angio

- 128-progressive Non pitting edema, no DM or HTN. Best Investigation?


A- venous douplex ✅ B- CT C- MRA

- 129-Male, diabetic, HTN, came with ulcer in lateral left malleolus for 3 months, (((pulse is
intact)))?
A. MRA. B. Ct angio. C. Conventional angio. D. Venous duplex for left lower limb✅

- 130-Pregnant lady in 15 WK gest comes with marked upper & lower limp oedema. BP 150/95
What is your best management?

2019 ‫ من سنة‬11 – 10 – 9 ‫تجميعات قلوري تيم لشهر‬


A.complete evaluation ✅ B.atenolol

- 131-Diabetic patient present with leg redness increase with dependent position, on examination,
diminished pulse and cold skin What is the most likely diagnosis? Peripheral arterial disease ✅

- 132-Patient present with painful of cord like swelling of left leg and she has hx of vircous vein 20
years back Best initial treatment?
a) NSAID ✅ b) Celxan. c) Heparien

- Varicose veins Tx for therapeutic reason : Endovascular laser ablation


- Varicose veins Tx for(( cosmetic))) : Sclerotherapy

- 133-Pt w/t varicose veins c/o swelling and heaviness, no pain, pulse intact .. everything normal
NEXT?
A- Duplex US✅.

- 134-spider veinous case , what is the management? Endoscopic laser ablation


‫ الن‬spider vein ‫ عالمة على‬liver cirroais ‫ ومن مضاعفات‬cirrosis ‫ تعمل‬esophageial varicosis ‫و دوالي المريء هذه تعمل‬
‫ فعالج هذا الدوالي يكون ب‬، ‫ نزيف‬laser ablation. . . .

- 135-Female presented with vascular malformation in the leg (hemangioma maybe) increasing. What
is the indication of removal? A-Pain✅✅ B-Ischemia C-Cosmetic

- 136-What is hard sign of vascular surgery?


A-Weak pulse B-Skin color change C-bruit ✅
the presence of hard signs mandates immediate action. The presence of hard signs of vascular injury
mandates immediate operative intervention.

- 137-Hard sign to detect vascular injury ?


A. multiple fracture B. change color C. decrease pulse D_pulsetile bleeding✅

- 138-Smoker elderly with preserved femoral and popliteal pulse put diminished dorsalis pedis, Mx ?
A. Localized thrombolysis B. Heparinization and observer✅

2019 ‫ من سنة‬11 – 10 – 9 ‫تجميعات قلوري تيم لشهر‬


❖ Compartment syndrome:
139-Patient with thigh hematoma, which of the following will indicate surgery? Pain✅.
‫ وهذا ممكن يسبب‬thigh hematoma ‫يقولك عنده‬
Compartment syndromes = Due to bleeding will compresstion to Tissue and blood vessle .
‫ طيب من‬، ‫ قبل ما تتكور الحالة ويحصل بتر‬، ‫ الزم فورا يروح للعمليات‬Compartment syndromes ‫ف لو ظهر اعراض و عالمات‬
‫ فالجواب هنا متى‬، ‫ وكمان تخديى وتغير اللون واشياء ثانية‬، ‫ هو الشعور بااللم ويكون الم شديييد‬Compartment syndromes ‫عالمات‬
‫اطلعه للع مليات لما يحس بااللم الن االلم من العالمات الهامة ل بداية حصول كومبارمنت سيندوم‬
Key word: common signs of compartment syndrome *5ps*
Pain, pallor, parenthesis, pulselessness, paralysis
- 140-Patient came after limb trauma, with severe pain and paresthesia between his toes and it was
pale. X ray shows fractures, intra compartmental pressure was 35mmhg what to do?
A. Internal fixation B. closed reduction C. external fixationn with 4 fasciotomy✅
Key word: normal intra compartmental pressure =0-8 mmHg
Above 30 indicate compartment syndrome and ttt with fasciotomy

141-pt involved in RTA stable, but there is left leg swelling and paresthesia between toes xray showed
fracture tibia pressure in posterior leg compartment is 35 mmHg :
external fixation with multiple fasciotomies ✅✅

- 142-30 years old had trauma which resulted in fracture of his right tibia and fibula After a while he
started to complain of numbness and severe pain within that area What would you do?
A.Heparin B.Embolization C.Fasciotomy ✅✅

❖ Testicular
- 143-Infant with Bilateral non tender scrotal edema and redness extending to groin. What's the
diagnosis?
A. Testicular torsion B. Epididymo-orchitis
C. Edema of testicular appendages D. Idiopathic testicular edema✅
Key word : infant = idiopathic
- 144-Patient came with testicular enlargement after an exercise what will you do for him? US
Key word: after exercise = testicular torsion =US
- 145-decrease in left testicular size post hernia repair ?
A. Testicular artery occlusion B pampiniform plexus occlusion✅

- 146-Pain with absent cremasteric reflex: A- Surgical consult ✅ B- US C- Culture


‫ وهذه حالة طارئة الزم فورا‬testicolar torstion ‫ ايش اول شيء يجي في بالنا؟؟‬: absent absent cremasteric ‫نشوف بس كلمتين‬
.‫جراحة‬
- 147-5 years old child found to have one testes in the scrotum and the other in the ingunal area what
to do? A-orchiopexy ✅ (( surgery to move an undescended (cryptorchid) testicle into the scrotum))

- 148-Pt recent abdominal surgery presented by fever&otheres. what is the most cause?
A-sepsis B- subphrenic abscess✅
(Other names. Subdiaphragmatic Abscess. Specialty. Infectious disease, gastroenterology.
Subphrenic abscess is a disease characterized by an accumulation of infected fluid between the
diaphragm, liver, and spleen.)

2019 ‫ من سنة‬11 – 10 – 9 ‫تجميعات قلوري تيم لشهر‬


- 149-female patient complains of urinary dribbling, dyspareunia, dysuria. What is the most likely
diagnosis?
A. Overflow incontinence B. Urethral diverticulum✅ C. Stress incontinence
A urethral diverticulum occurs when an unwanted pocket or sac forms along the urethra.
Key word: Dribbling = urethral deverticulum
- 150-year sold boy brought by his parents, he has nausea, severe vomiting for 20 minutes
and now semi comatose. The parents mentioned that he has same episode two weeks ago
for 5 minutes without deterioration in consciousness. On examination there is right testicular
mass that does not transilluminate with light. What is the best action to do ? Surgical
exploration. ✅
Keyword: testicular torsion
- 151-Testicular pain absent cremaster reflex (for 4hours) , clinical pic of (torsion) what to
do?
Surgical exploration ✅

- Appendsitis
- 152-Female (obese) with typical appendicitis: Right iliac pain and tenderness Nausea and vomiting.
Loss of appetite. Leukocytosis What is the proper management:
A. Abd ct ✅ B. Abd us
Key word:
Adult=CT
Child and pregnant = US

- 153-Post appendectomy female came with LR abdomen mild tenderness Ex Normal By CT there is
2*2 collection in Retrocecal :A. Exploring laparotomy B. percutaneous drainage
C. laparoscopic D. conservative with Anitbiotic✅
5 ‫ فقط واالبحاث قالت انه امن وفعال بينما لو اكثر من‬antibiotic ‫ سم نختار‬5 ‫لو اقل من‬
antibiotic ‫ = مع‬percutnus ‫نختار‬

- 154-Pathophysiology of appendicitis in 58 male ?


A. change in blood distribution B. peripheral vasoconstriction ✅ C. decrease heart index

- 155-Patient obese came complaining of rt iliac fossa and tendrenss : CT abdominal ✅HE IS
OBESE

- 156-Young unilateral testicular swelling since 1 day, on exploration viable but cord edematous :
A-Torsion B- Incarcerated inguinal hernia C- Appendicular torsion✅
2019 ‫ من سنة‬11 – 10 – 9 ‫تجميعات قلوري تيم لشهر‬
Key word: unilateral and cord edematous
- 157-Post appendectomy day-4 presented with abdominal pain and feculent discharge most
appreciate step? A. IV antibiotics✅ B. Ex lap
.‫ يعني تجمع بكتيريا اهم اهم خطوة والزم هو نعطيه مضاد حيوي‬pus ‫يقولك عنده‬

- 158-Case scenario ....appendicitis but not ruptured( best )investigation:


a. CT abd✅ b. US abd c. Exploratory
key word:
adult= CT
child and pregnant= US
- 159-Peds 8 yrs old with RLQ pain and rebound tenderness what's confirmatory test?
A. US abdomen ✅ B. MRI abdomen C. CT abdomen

-
- 160-Post appendectomy came with mild pain and collection 2x2 Asking about management:
A. Per cutaneous drainage B. Conservative with Abx✅✅
5 ‫اقل من‬
- 161-Post-appendectomy, 0.5 cm carcinoid found on the tip of the appendix, mx?
Nothing = If carcinoid tumor less than 2cm > appendicectomy only
More than 2cm or at base of appendix> right hemicolectomy

- 162-After appendectomy dr found a carcinoid mass more than 5 cm on the tale of appendix Next?
C.T scan abdomen and chest for staging✅✅

- 163-Most surgical emergencies in pediatrics ? Appendicitis ✅✅

- 164-Old age came with symptoms if appendicular mass and treat it What you will do?
A. Colonscopy 6 weeks ✅ B.12 weeks
‫ أسابيع النو كبير في العمر‬6 ‫هنا احنا شلنا الماس فنعمل كولونسكوبي بعد‬
- 165-after initial conservative treatment of appendicular mass :
A.Open appendectomy after 12 weeks B.Laparoscopic appendectomy after 12 weeks ✅

- 166-Appendicular abscess 15cm*17cm in a 33 y old man. How to manage ?


A. antibiotics ✅ B.Surgical
(No drainage in the options)
Keyword: to ttt abscess first start with drainage and then give antibiotics

- 167-During app. Surgery appendix not find !? How to find it = Follow tenia colli

- 168-pt febrile , abdominal pain , rebound tenderness at mcburney's point , wbc high , x ray show
right fossa ?
A- appendectomy✅✅ B- gall bladder US C- urology consultation
All sings of appendicitis are present(alverado score)
Less than 6 = observation

More than 6= surgery

2019 ‫ من سنة‬11 – 10 – 9 ‫تجميعات قلوري تيم لشهر‬


-
- 169-After open appendix, seroma collection and leak from gap of sutures , no inflammation, what to
a-Dressing✅ b-Evacuation c-Ab
Minor, small seromas don’t always need medical treatment. That’s because the body may naturally
reabsorb the fluid in a few weeks or months.---the seroma if it’s large or painfu = Drain

- 170-Post appendectomy case 1 week , not complain ,but upon exam there was seroma with in
gaping wound .what u will do ?
A. repeat and care of the dressing✅ B. drainage C. ct abdomen

- 171-Perforated appendicitis post laparotomy, presented with fever, abdominal pain CT done
showed foreign material , reexploration done, gauze was found intraabdominal ،intraabdominal ،
What to do? A-Issue complaint against assistant B-Call your lawyer and report the incident
C-apologize to the patient and tell her what happened ✅

- 172-Young male with right lower abd pain and tenderness, low grade fever On abd xray:
radioopaque material (small at iliac crest site) What to do?
-urgent uro consultation-non contrast CT -appendectomy ✅
Keyword: Clear case of appendectomy
- 173-Case of acute appendicitis, fecolith and abscess seen on CT management?
- open Appendectomy and drainage
-lap appendectomy and drainage✅
-percutaneous drainage
.‫ تكونت وهي تحصل نتيحة التهاب الزائدة‬stool ‫يقولك كان فيه قطعة‬
.‫ الزم نطلعها وبعدها نعمل العملية إزالة الزائدة‬Drain ‫يفضل اول شيء‬
- 174-After laproscopic appendectomy . Pt come with retrocecal collections. ?
A. surgical draineg B. percatenous drainage ✅✅
Key word: when to choice endo scoop drainage if pt have discharge but clear and there is no symptom of
infection as fever
When to choice prectenous drainage If pt have leukocytosis or abscesses or fever (signs of inflammation)

175-Appendicitis case with abscess how would u manage it :


A.open B.laparoscopic C.antibiotics D. percuatanous drainage ✅

-176-8 days Post appendectomy with abdominal pain, distention, inability to pass stools, In exam:
feculent material discharge with signs of peritonitis?
A- CT B- IV antibiotic C- Exploratory laparotomy✅
Keyword : peritonitis = laparotomy
❖ Benign Prostatic Hyperplasia & Prostate
- 177-A 73-year-old man presents pain in his right thigh. This has been getting progressively worse
for the past 9 months despite being otherwise well. An x- ray is reported as follows: X-ray right
femur Radiolucency of subarticular region suggestive of osteolysis. Some areas of patchy sclerosis
Bloods tests show: Calcium 2.38 mmol/l Phosphate 0.85 mmol/l Alkaline phosphatase 544 u/L
Prostate specific antigen 4.4 ng/ml. What is the most appropriate action?
A. Vitamin D supplementation
B. Check serum testosterone
C. Referral to an orthopaedic surgeon
2019 ‫ من سنة‬11 – 10 – 9 ‫تجميعات قلوري تيم لشهر‬
D. Referral to a urologist ✅(could be prostatic Ca mets to the bones.)
E. IV bisphosphonates

- 178-75 y male c/o(( back pain, difficulty passing urine)), psa: 84(high) , ALP: 410 ? (high)
A. prostatitis B. prostatic cancer ✅ C. urinary bladder ca D. BPH
**High ALP indicates bone metastasis.
Key word = back pain and difficult passing urine
Any pt with high PSA and old =prostatic cancer
And treated with Alph blocker and calcium channel blocker
- 179-80 y old man with bilateral hydronephrosis on Us images ?
A. Prostate enlargement / Ca B. Bladder ca > come more aggressive pain, hematuria
C. urethral stricture D_benign prostatic hyperplasia ✅
Keyword: cause of hydronephrosis cancer as(bladder- cervical – colon – prostate )
- 180-Part of urethra affected by surgical (trauma) in males ?
a. membranous b. penile c.Bulbar✅
which lyer inj during catheter = membranous
‫في اإلجابات ماحصلت بي على طول اختار سي‬
N.B: Membranous for pelvic fracture after MVA

- 181-Old patient with loain pain . Us shows biliteral hydronephrosis? enlarged prostate
Keyword: hydronephrosis = swelling in kidney most common cause is cancer

- 182-By Cyctoscopy: Redness in the dome of bladder What is the most likely diagnosis:
A-ischemic colitis B-diverticular disease. C-Transitional bladder cancer✅
Keyword; redness in dome of bladder
- 183-Old M c/o urinary sx, prostate median lobe hypertrophy, what is best for this pt?
Annual Prostate-specific antigen ✅
.‫ هذا انتيجين يرتفع لما يكون فيه ورم في البروستات من اسمها‬:high Prostate-specific ‫للعلم‬
Keyword: prostatic hypertrophy
- 184- Old pt came with difficulty urination and low back pain with high Prostate-specific antigen
dx?!
A- Prostatic cancer✅✅.
Benign prostatic ‫خذوا هذا السر ( اي احد كبير في العمر وجاء يقولك عنده الم في اسفل الظهر وكان عنده تاريخ سابق ب‬
(metastasis ‫ وسار له‬prostata cance ‫ نشك على طول انه عنده‬difficult urination ‫او قالك من االول وانا عندي‬hyperplasia
.‫ فيجيك ب الم في الظهر‬spinal ‫ واكثر اكثر مكان يروح له ل مرض سرطان البروستات هو‬، ‫ يعني بدء السرطان ينتشر‬spinal ‫)لل‬

- 185-Urethral trauma with bleeding post MVC. Next step?


Suprapubic catheter✅✅
‫ماينفع نختار فولي كاث الن فيها اصابة ماينفع ندخل القسطرة واليوريثرا ماهي سليمة‬
Keyword: retrograde urethrogram is essential for diagnosis of urethral injury, or urethral stricture

- 186-75yeard old male with ((( back pain ))) and urinary symptoms (obstructive) ,ALkaline
phosphatase hight and (((PSA 80))) what is the most likely Dx?
-BPH. -prostatic cancer ✅✅
** PSA = Prostatic specific antigen = if above 4 = ‫معناها سرطان الن هذا انتيجين مايرتفع اال في االورام البروستاتا‬

- 187-65 yrs came with mild decrease in urination , us showed median lob hypertrophy of prostate ,
PSA and digital rectal was normal , urine analysis and renal function normal what to do?
2019 ‫ من سنة‬11 – 10 – 9 ‫تجميعات قلوري تيم لشهر‬
A. annual renal function tese ✅ B. Periodic investigation of PSA
C. Beta blocker D. Cystoscopy
Normal PSA =0 – 2.5 normal

❖ Hemorrhoids
- 188-sclerosing patient with history of rectal bleeding, anoscopy show swelling at 3,7 o'clock,
sclerosing thera
py is planned what is the most appropriate to do sclerosing therapy
A. external hemorrhoids
B. internal hemorrhoids✅✅
C. prolapsed hemorrhoids
D. thrombosed hemorrhoids
Keyword: sclerosing = internal hemorrhoids

- 189-post hemorrhoidectomy pt has urinary retention. Cause:


a. Trauma b. Anesthesia affect✅

- 190-Patient after hemorrhoids operation develop Suprapupic pain with inability to pass urine what
is cause? Inadequate analgesic

- 191-20s years old presented with pain during defecation minimum amount of blood, on
examination the doctor seen a posterior midline fissure. He couldn’t do PR (rectal examination)exam as the
patient was in severe pain Vitally stable What’s your next step? A. EUA (Examine under anasthesia EUA)

- 192-Old patient who has constipation on and off with streakin of blood in the stool with no fulness
in the rectum ( no mention of pain )?
A. Sigmoid cancer
B. rectal cancer ✅✅ old age
C. chronic hemorrhoid.

- 193-Old pt-fatigue sign of anaemia hb8 , stools with streak of blood -has hemorrhoids stage 2 ?
A. Sigmoid ca. B.rectal ca✅✅ C.chronic hemorrhoid
‫ لو انت جراح‬، Rectal cancer ‫ الزم الزم ما تستبعد وجود‬hemorrhoid ‫ اي مريض يكون كبير بالعمر وعنده‬: ‫سمعوا هذه المعلومة‬
cancer‫ كويس الن كثير بنالقي صدفة عندهم‬rectum area ‫ الاازم بالمرة تفحص‬hemorrhoid ‫وجاك مريض ((كبير بالعمر )) وعنده‬
** Hemorrhoids produce the symptom of pruritus (itching) in the rectal and/or
anal area while rectal cancers usually do not..

- 194-Patient came with painless blood after defecation, whats the diagnosis?

2019 ‫ من سنة‬11 – 10 – 9 ‫تجميعات قلوري تيم لشهر‬


A. Abscess B. Fistula Hemorrhoids✅
Keyword: Hemorrhoid = painless
Anal fissure= pain full with defecation
- 195-Sclerotherapy use in management ?
A. Internal hemorrhoids ✅ B.External hemorrhoids
‫ = سكليروا = سكين = الزم يدخل جوه‬internal
- 196-Hemorrhoids type 4, what is the management: Hemorrhoidectom

- 197-case of hemorrhoid. what is the Indication of sclerotherapy? A- Internal✅. B- Externa

- 198-Patient with (perianal) painful (swelling)-vitals normal, (wbc normal)?


A-pile. B-perianal abscess C(-perianal) hematoma✅. D-anal fissure

- 199-Anal fissure with sentinel pile not responsive to medical therapy. Next step?
A)_Lateral external sphincterotomy. B)_ Lateral internal sphincterotomy(LIS) ✅
n . ‫ مع‬n ‫ ( = حرف‬Anal = internal ) ‫ربط‬
Lateral internal sphincterotomy is an operation performed on the internal anal sphincter muscle for the
treatment of chronic anal fissure. The internal anal sphincter is one of two muscles that comprise the anal
sphincter which controls the passage of fece

- 200-streaks of blood after defecating and pain? Anal fissures ( lateral Sphincterotomy)✅
*s*treaks of blood = fi*ss*ures
‫ من اسمها قال‬sreaka ‫ يعني تمزق‬fissure

- 201-pt with intermittent perianal pain and discharge, on P/E theres low-lying fistula with an opening
in posterior wall above anal verge. What to do?
A. MRI B. Fistulogram C. Fistulotomy✅ D. Lateral sphincterotomy
....‫ وعالجها نشيل الفيستوال‬fistula ‫واضح انه عنده‬
If want treated = fistulotomy
If want investigation=fitulogram
- 202-Perianal pain on examination no lesion, finger exam showed internal swelling with purulent
discharge on the anal canal , Dx?
A. perianal hematoma B. internal abscess✅

- 203-Male adult with perianal swelling and discharge, then ruptured?


A. Anal fissure B. hemorrhoid C. anal abscess D. fistula✅
Discharge ‫ و‬swelling ‫ وتعمل‬perianal ‫فستيوال غالبا تكون مكانها‬

- 204-Pt with anal abscess then you found fistula on examination draining pus , the opening is in the
posterior rectum wall: management:
A. MRI B. fistulography C. fistulotomy/ectomy ✅

- 205-31 year old male with constipation and during perianal exam there was sever pain and linear
laceration at 6 and 12 oclock whats the dx:
A. Anal fissure ✅ B. anal fistula. C. hemorrhoids
Key word = 6 and 12 oclock
- 206-pt with bleeding after defecation ( painless ) ?
2019 ‫ من سنة‬11 – 10 – 9 ‫تجميعات قلوري تيم لشهر‬
A. Anal fissure B. Hemorrhoid ✅

- 207-pt presented with 3 months bleeding after stool, a perianal mass, and weight loss. Histology
confirmed adenocarcin oma. CT finding shows no lymph node involvement and mass limited to
3cm from anal verge. What's treatment modality?
A. Radiotherapy B. Abdominoperineal resection✅
C. Chemotherapy D. Low resection only
Less than 6cm from anal verge > Abdominoperineal resection
More than 6cm > Lower anterior resection.

- 208-Case of perianal swelling , per rectal bleeding , investigated : it is 1 cm from anal verge biopsy:
adenocarcinoma (rectal) Mx:
A-Abdomen perineal resection ✅ B-low anterior resection C-chemo/radio

- 209- Adenocarcinoma of colon, 1cm from anal vege?


A. low anterior resection of rectum (LAR) B. Abdominoperineal resection✅

- 210-patient with anal swelling and discharged, swelling is rapture, No fever:


A- fistula abscess✅ B- thrombosed pile

- 211-Anal itching and pain post defecation and ask ddx?


A. internal hemorrhoid B. Anal fisure ✅✅ ‫يعملوحكة‬
Key word =pain with defecation

212-Anal fissure failed medical Treatment? lateral internal anal sphincterotomy

- 213-Anal fissures with skin tag not respond to drug, want procedure?
A- Internal sphernctomy✅ B- External sphrenctomy C- Anal curettage with remove tag

- 214-Post partum woman with painful defecation and bleeding and pain after defecation; what is the
cause?
A. anal fissure ✅ B. Haemorrhoids C. thrombosed pile

- 215-pt with Anal dischsge and he gave history of ruptured anal abscess :
Anal fistual ✅. Anal' fissure Anal piles
Key word: =rupture = anal fistula
Boggy = abcess
- 216-Patient have pain with and after defication and some blood with it diagnosis?
A-hemorrhoids B-anal fistula. C-anal fissure‫✅ التمزق يعمل الم‬ D-abscess

❖ Wound
- 217-First evaluate surgical wound by : inspection
Key word:
- If there’s signs of infection* (pulurent discharge, redness, tenderness) then wound exploration
if superficial > clean, dressing and you may give abx
- If deep collection suspected >* order CT
- If small collection less than 4cm >* abx
2019 ‫ من سنة‬11 – 10 – 9 ‫تجميعات قلوري تيم لشهر‬
- If large 4cm and more* > percuteanous drainage + abx
- If signs of peritonitis > Laparotomy

- 218-10 cm laceration in anterior thigh what to do?


a-Tourniquet b-Compress ✅ c-Vascular consult d-Band or tourniquet above femoral artery
)‫انتوا تخيلوا الموقف لو اصابكم اي جرح كبير ونزل دن كثير ايش اول شيء تسوونه ؟؟ اول شيء بتاخذون قماش وال منديل ( تضغطون‬
.‫على الجرح عشان يوقف نزيف‬
- 219-Elderly pt sacral ulcer manag?
A-Depriment and skin graft(if necrosis) B- Primary closure C- Daily dressing✅

- 220-Elderly pt underwent for a major surgery ..he need blood transfusion 15 PRBCs .. after that he
start bleeding from wounds, nose, from NGT (stomach) .. what is the cause:
A- vWBD. B- thrombocytopenia✅. C- hemophilia. D- no DIC in choices

- 221-25 years old with perforated appendix did laparotomy, after 5 days wound is tender and pus
discharge what to do:
a- wound dressing
b- b-exploratory laparomtmy.
c- c-iv abx
answer is: wound exploration

- 222-Girl with stab wound in the right anterior axillary line below the costal margin, stable?
A. Observation.
B. immediate surgical exploration. Perform FAST
If pt stable and chooses have CT go with it. If not choose A,
if unstable Go with CT if not do Fast to pt.
- Neck stab wound
• Zone I, and III > C.T angio
• Zone II asymptomatic > C.T angio
• If symptomatic> surgical exploration

2019 ‫ من سنة‬11 – 10 – 9 ‫تجميعات قلوري تيم لشهر‬


- 223- Pt with neck injury in zone I, vitaly stable, what’s next :
A. neck exploration
B. CT face and neck
Answer is: A

- 224- Neck injury->pt had Subcutaneous emphysema_> next step?


A. Ct neck
B. explore neck
Answer is: A

- 225- Neck trauma stable anterior to the angle of the mandible Manag ?
A. CTA
B. exploration
Answer is: A

- 226-18 yrs with stabbing wound 10cm of thigh what next step management:
A. compress on wound
B.tourniquet upper thigh
Answer is: A

- 227-Neck trauma in area retro auricular above the angle of the mandible (area 1 was
not mentioned), patient stable and oriented. Most appropriate step?
A. CT angio.
B. Neck exploration✅
Key word: zone 1 and zone III=CTA angio
Zone II= if asymptomatic = CTA angio
Symptomatic= surgical exploration

2019 ‫ من سنة‬11 – 10 – 9 ‫تجميعات قلوري تيم لشهر‬


- 228-Patient has stab wound affect the duodenum and vital signs are stable what is your management
?
A. Ct scan
B. Laparotomy
C. Conservative
Answer is: A

- 229-pt with forearm fracture and open wound 1cm , what’s TTT :
A. close reduction.
B. wire.
C. cast
D. debridement, irrigation, fixation✅
Answer is: D

- 230-Pt post-operative and leak 20ml fluid from the wound:


A. Dressing B. Wound exploration✅ C. Lap
Key word: any purulent or redness or tenderness do exploration
- 231-Post open appendectomy case with pain in wound site on examination u see pus oozing from
site of surgery what will u do next?
a. percutaneous b. US abd c. iv antibiotics✅ d. open surgery
key word :puss = oozing = anti
puss with abscess = open surgery
- 232-Pt has infected wound has pus come out from it what next step?
A-Inspection B-exploration of wound✅
Key word : first step exploration
Then give anti

- 233-After herniotomy surgery for 5yrs boy came with fever and pus discharge and part of mesh
seen ? A.give iv antibiotics B.draining of pus C.draining of pus and remove mesh ✅ D.
observation

- 234-X Ray showing both distal ulna and radius fracture with volar displacement On examination a 1
cm wound was seen at the volar aspect of the wrist Asks about initial management:
A. Closed reduction with above elbow cast
B. Closed reduction with below elbow cast
C. Irrigation of the wound and Closed reduction with below elbow cast✅

2019 ‫ من سنة‬11 – 10 – 9 ‫تجميعات قلوري تيم لشهر‬


- 235-Young male with MVA came by ambulance to ER conscious, with thigh wound covered with
sucking gause : call surgeon in duty

- 236-Trauma patient with a wound on his thigh subcutaneous fat is lost vasculature underneath is
exposed what provides the best management :
A. Debridement with primary closure
B. Primary repair
C. Debridemnt with secondary closure✅
D. Debridement with vacuum assisted closure
Key word: fat and vascular lost = secondary closure
If crash wound = primary
- 237-25 year old male Pt 8th day post surgery with wound site redness & tenderness with purulent
discharge.. most appropriate?
A. IV antibiotics B. CT abdominal C.open drainage✅ D.exploratory laparoscopy

- 238-Patient post surgery, there surgical wound redness, tenderness with no discharge, abdomen is
soft and lax .what to do? A. Antibiotic ✅ B. wound drainage C. CT abdomen

- 239-Patient post surgery, there pus coming from surgical wound with tenderness and leukocytosis,
abdominal examination was done it was soft lax with no tenderness all over, what to do?
A. Antibiotics ✅✅ B. wound opening C. CT abdomen
open...‫ نختار‬Fever ‫ والمريض عنده‬absess ‫ اذا قال فيه‬Antibiotic ‫ نختار‬pus or ooze ‫اذا قالكم‬
Key word: leukocytosis signs of infection = anti
If ask about management = prectanous dringe

- 240-Patient was hit with wood 5 days ago, now he comes with severe RLQ pain. On examination
there is a small opening with pus discharge, put when tried to extend his thigh there was severe pain
and you couldn't move it. What to do?
A. CT✅ B. wound drainage C. antibiotics

- 241-Patient post resection and colostomy presents with spiking fever for 1 week. Chest is clear.
Abdomen is clear. Wound is clear. Fever 38.2 Next step?
A- CT abdomen ✅ B- Reassure
Key word: spiking fever there is sings in peritonitis problem = ct
- 242-person with car accident on left thigh show neurovessle fat and tissue out mange ?
debridment and skin grafting

- 243-k/c HTN with lateral mules ulcer, next appropriate management?


A- wound examination ✅. B- duplex. C- Ankle brachial indexes

- 244-Pt was stabbed by knife into his neck (ant to the right ear way through his mandibule )
Hé is stable with 1 or 2 cm wound ozing blood What is the (next) step in managment :
A-CXR. B-C.T angio. C-wound exploration ✅
‫ نشوف هل فيه اي‬، ‫ نسوف عمقه‬، ‫ انه نفحص الجرح‬stap wound ‫ اول شيء نعمل ل‬next step , ‫طيب هو السؤوال يقول‬
c t‫بعد الفحص نوديه لل‬forgen body

- 245-Patient with perforated appendicitis after surgery had pus from wound, pain localized to the
surgical site. No guarding no fever what best initial treatment is:
2019 ‫ من سنة‬11 – 10 – 9 ‫تجميعات قلوري تيم لشهر‬
A. Antibiotics✅✅ B. Open drainage. C. Imaging guided drainage D. Wound exploration
‫ مازال مستمر‬pus ‫ لو ال‬culture ‫نركز هو قال اول شيء نسويه والمصادر تقول نعطيه مضاد حيوي ك اول خطوة مع اخذ مزرعة‬
. . ‫ وخااصة لو المريض قالك عنده حرارة‬، ‫وماراح بالمضاد الحيوي الزم افتح الجرح واشيل الغرز‬
You may be started on antibiotics to treat the surgical wound infection. The length of time you will
need to take the antibiotics varies, but will typically be for at least 1 week. You may be started on IV
antibiotics and then changed to pills later. Take all of your antibiotics, even if you feel better.

- 246-Patient post surgery presents with disch'rge from middle of the wound. Next step?
- Daily dressing. - Wound inspection✅✅ - Wound explor'tion
‫ اول خطوة في اي فحص الاازم اول شيء‬inspection

- 247-Patient with right lower chest stab wound. Fast showed free abdominal fluid. What is your next
step? A. exploratory laparotomy ✅ B.thoracentesis C.chest tube
- 248-Patient with stab wound what to do next ? Local wound exploration✅

❖ Radiology
- 249-An elderly with IHD day 2 post cholecystectomy presented with sudden chest pain SOB and
vitally hypotension and tachycardia whats best ?
A. CXR B. ECG ✅ the best C. CT angio D. LL duplex US

- 250-Elderly with weight loss (10kg) and anorexia in endoscopy there is large ulcer in stomach ,
biopsy was taken and the result is invasive adenocarcinoma . What is the next step:
A. endoscopic US B. CT abdomen and chest and pelvis✅ ‫عشان اشوف الورم انتشر ال ال‬
C. abdominal US

- 251-Best diagnostic method for pulmonology embolism?


A. D- dimer B. Spiral CT✅
Key word: pulmonary angiography : best definitive diagnostic test
Confirm D dimer for chronic pulmonary embolism

- 252-Chronic thromboe-embolism confirme :


A. QV ratio ✅ B. spiral CT C. D dimer

- 253-supraclavicular LN met what you do for primary site : Gastroduodenoscopy

- 254-How to diagnose Adhesion post operative?


CT, but Initially > abdominal X-ray✅ (Air-fluid level, Bowel dilatation)
Key word: initial x-ray
- 255-Old patient with worsening of his constipation, labs shows positive occult blood in stool. Colon
cancer suspected what to do?
A. Colonoscopy✅ B. sigmoidoscopy C. CT abdomen
-
- 256-Pt melena and fresh blood per rectum, upper and lower endoscopy negative, next investigation:
Capsule endoscopic
- Key word: endoscopic positive
- ‫ نستعمله لما نكون‬، ‫الكابسوال هذه فيها كاميرا صغيرة ندخلها للجهاز الهضمي ويمشي على كل الجهاز الهضمي ظن بدايته لنهايته‬
‫ عملنا‬endscope ‫ فنلتجا للكبسولة هذه‬. ‫ ومالقينا سبب النزيف‬.
2019 ‫ من سنة‬11 – 10 – 9 ‫تجميعات قلوري تيم لشهر‬
- 257-Old Man with falling trauma CT brain is normal but he is complaining of bilateral foot pain:
Check pulse > analgesia > X-ray
Key word: this case of fracture
- 258-best diagnostic for Coarctation of the aorta for neonate ? echocardiogram is the most
commonly used test to confirm the diagnosis ( Not ultrasound or C.t ) in adult also use echo
Key word: confirm= echo
Initia =US
- 259-A man who is a known case of diabetes presented with hemiparesis 15 hours after some
procedure ?
A. tpA B. Warfarin C. Ct angio✅
، ‫ فنعمل سي تي انجيوا‬small vessl brain ‫ الن قال عمل بروسيجر فممكن سار له ارتفاع شديد بالسكر واثر على‬strok ‫احتمال يكون جاه‬
blood vessel‫ عشان نشوف‬angio ‫اخترنا‬
Key word: hemiparesis or hemiplegia cause by hemorrhage or thrombosis and do ct angio
- 260-Pt with bronchogenic carcinoma, presented with progressive SOB, there’s elevated jvp, clear
lung and quiet heart sounds. What will confirm your dx:
A. CXR B. ECHO ✅ C. ECG
Key word: CT faster then MRI
This case of cardiac tamponed so diagnosis with echo = quit heart sound
- 261-Pt presented with stabbed wound after wound exploration you found anterior abdominal fascia
penetration, (his vitals were stable) what's your next step?
A. CT abdomen ✅
B. MRI abdomen
C. Exploratory laparotomy
D. Diagnostic laparoscopy = definitive diagnosis
Key word: any pt with abd trums below 4 inter costal spaice = exporatory lab
- 262- Best way to clear show cervical spine:
A. Clinical judgement or assessment. B. Lateral cervical x-ray✅
- 263-child episode of PR bleeding, fresh ; black stool (have both melena and hematochezia) ; NGT
showed greenish fluids ; colonoscopy was negative what is next to detect source of bleeding ?
A. upper endoscopy. B. Tc 99m✅. C. Barium. D. US
- 264-Chlid overweight with X-ray? slipped epiphyseal plate

-
- 265-Old with back pain radiated to back , CXR air under diaghram ? perforated peptic ulcer
- Key word: air under diaphragm
- 266-Pt in ICU with coffee ground vomit? Stress gastritis

2019 ‫ من سنة‬11 – 10 – 9 ‫تجميعات قلوري تيم لشهر‬


- 267-K/c of ulcerative on med presents with abd pain Invistigation show enlarged transver colon 15
cm or mm? Management? A.Total colectomy with ileostomy B.Proctocolsctomy with ileal
pouch C- steroid ✅
Key word : ulcerative colitis treat with steroid and then x-ray to detect colon perforating if show
perforating do laparotomy
There are many potential cause of coffee ground vomitus ancluding gastric ulcer – esophageal varices when
swollen vein in esophagus burst and bleeding gastritis irritation of the stomach lining cirrhosis sever
scaring on the liver and reduce liver function
- 268-30 something morbidly obese male , how to decide best reduction surgery he will have ?
A.Barium enema B. ct abdomen C. ultrasound abdomen D. GI endoscopy✅

- 269-farmer, wood brick in his abdomen 4days ago, 3 days ago he developed severe pain in the same
area, on exam there was green discharge coming out, what will you do?
A. take culture from the discharge.✅ B. CT abdomen. C. Drainage
key word: wound infection
- 270-Stap in abdomen right anterior axially line, below costal margin, patient stable conscious what
to do? a-Ct abdomen✅ b-Us FAST

- 271-cervical trauma in ICU patient after motor vehicle accident ? C.T

- 272-3 weeks post gastrectomy presented with progressive abdominal pain increasing in intensity.
What is the next appropriate investigation? 1_US 2_ X-ray 3_ CT ✅ 4_Laparotomy
Key word: 3 w post gastroctomy with progressive abd pain and nausea (adhesive ) CT
If day reassurance
- 273-Patient complained of abdominal pain 3 weeks after sleeve gastrectomy, what is the diagnostic
test?
1-US 2-CT with contrast✅ 3- Plain abdominal X Ray
Key word: diagnostic CT for perforation
- 274-Old male patient admitted as a case of large intestinal obstruction, underwent rigid
sigmoidoscopy which showed a mass in sigmoid region, biopsy was taken and resulted as
adenocarcinoma, what’s your next step? A. Colonoscopy ✅ B. CT abdomen C. MRI
pelvis D. Sigmoidectomy
- Key word: mass in sigmoid

- 275-CT in truma important for? retroperitoneal injury✅

❖ Spleen Laceration
- 276-Splenic laceration and thoracic aortic injury. You want to transfer the patient. What is
important?
A. Transfer him to laparotomy B. Transfer him to thoracotomy ✅ C. Angiogram
Key word: transfer pt and thoracic aortic inj

- 278- Post splenectomy came complaining of left side pain, Reduce air entry in the left side, Dx?
A. Subphrenic abscess ✅✅ B.Post splenectomy overwhelming syndrome

- 279-RTA patient, presents 2 days later with ascending aorta injury and splenich laceration, the best
immediate management is:
2019 ‫ من سنة‬11 – 10 – 9 ‫تجميعات قلوري تيم لشهر‬
A.Take the patient for thoracotomy B. Take the patient for laprotomy ✅ C.Call vascular
surgeon
Key word :choice B if after 2 day and pt bleeding
Choice C if pt RTA and transfer to another hospital
- 280-Patient after pacreatitis episode develops upper GI bleeding picture , scope was done,
gastric fundus bleeding was found ,sclerotherapy done Duplex ultrasound showed: splenic vein
thrombosis with patent portal vein whats is your management: A-Splenctomy ✅
- 281- 12 year old received a nonspecific blunt trauma on his abdomen and later presented with
generalized abdominal pain. Imaging of the spleen showed a 7mm hematoma and 4 cm tear(grade
3). Your management:
A. splenectomy B. Spleen preserving surgery ✅ C. Conservative

- 282-Spleen injury 1? consertive treatment

- 283-Spleen injury 3? partial resect. ( preserving surgery )


Key word: 1 and 2 = conservative treatment and 3 = preserving surgery and 4 = remove
- 284-4 cm tear with hematoma 7cm grade 3 ? A. splenectomy B. splenic conservative surgery✅

- 285-Laceration spleen grade 4 ? Splenectomy

- 286-You are in (( small hospital or something )) and you have a patient with aortic injury and spleen
laceration grade 3 or 4 and you want to transfer him , how do you call ?
A.ICU B-Vascular surgeon C. General surgeon
Key word: small hospital and any pt with GIT send to surgeon
- 287-30 years old male after RTA had a splenectomy, what will be low after hours from the surgery?
A. insulin✅ B. glucose C. vasopressin ‫الن فيه جزء من البنكرياس الصق في السبليين‬
Key word: any pt after splenctomy increase blood sugar
Surgery and anesthesia cause release stress hormone these hormone make the body less sensitive to insulin
which may result in elevated blood sugar
- 288-Patient child with fever , abdominal pain and splenomegaly and the spleen is tender what
culture is important ?
A/Single blood culture B/Multiple blood cultures✅ C/bone marrow aspirate culture
most likely typhoid: Bone morrow as most Sensitive, but invasive so multiple blood culture better .

- 289-If splenectomy > pneumococcal, HIB, meningococcal > given 2 weeks before the surgery of
splenectomy✅✅
‫و االنفلونزا احتمال كبير جدا بعد‬meningitis ‫ للنيمونيا و ال‬vaccin ‫الن ازالة الطحال يقلل المناعة جدا جدا لو ما اعطيناهم هذا ال‬
‫ سبحان هللا‬spleen ‫ عشان تعرفوا نعمة‬، ‫عملية ازالة الطحال يصابوا بهذه العدوى بسهوووولة‬

- 290-Patient 3 days post splenectomy develop fever 38.5 what is the cause of bacteremia?
A- UTI ✅ B- Peritoneal transfer
Key word: fever day 3-5 post op is UTI.

- 291-Pt postoperative day 3 has gram negative bacteremia how it rech to blood ?
A-translocation B-UTI✅✅ C- gut
Key word: 0-2 day = atelectasis or pneumonia
3-5day= UTI
5-7=DVT
2019 ‫ من سنة‬11 – 10 – 9 ‫تجميعات قلوري تيم لشهر‬
7 day = wound infection
8-15= drug fever or deep abscess

- 292-Alcoholic present with hematemesis. Imaging shows splenic vein thrombosis. Management?
Splenectomy✅
.‫ هذا العالج الوحيد لهذه الحالة‬Splenectomy . ‫ على طول اكتب العالج‬splenic vien thrombosi ‫اول ما نشوف كلمة‬
Key word: splenic vein thrombosis = splenoctomy
❖ Hernia:
- 293-Woman in her fifties asymptomatic has a small femoral hernia, what is the best management:?
A. Observation B. Open with mesh C. Simple open D. Laparoscopic repair✅
*Femoral hernia*= We prefer to repair a femoral hernia *laparoscopically* because of its ease
of access.
Old people > femoral we do open
Key word: any femoral hernia = repair
- 294-Solder with bilateral inguinal hernia ? Lap w mesh
Key word: bilateral + solder = lap with mesh or lap open with mesh
- 295-30 year-old has a large indirect inguinal hernia sac that is reducible with mild effort. Bowel
sounds are heard in the hernia sac. This hernia is the result of which of the following?
A. A defect in the abdominal wall with protrusion of abdominal contents through the internal inguinal
ring, canal and external ring
B. A traumatic defect in the abdominal wall
C. A congenital defect in the abdominal wall allowing protrusion of abdominal Contents through the
inguinal canal✅
D. A defect in the abdominal wall caused by constipation

- 296-Pt 56 c/ o sudden RLQ pain after lifting heavy object ; and mass in RLQ ; Cough negative ; abd
muscle tense even with clenching . He is on anticogulation due to A fib . What the most apporpieate
mgx ? ^ here I am thinking of Rectus sheath hematomea == rest and anaglesic✅

- 297-Man developed rt abd swelling after lifting heavy object, painful irreducible , nocough impulse
..dx?
A. Ventral hernia B. hematoma✅
2019 ‫ من سنة‬11 – 10 – 9 ‫تجميعات قلوري تيم لشهر‬
- 298-hernia, how to manage:
A- laparoscopic repair with mesh ✅
B- open repair with mesh + same options without mesh!

C-

- 299-pt did herinal repair with mesh 12y ago and came by obstruction sign and no sign hernia repair
?
A. Adhesion ✅✅ B. Late onset crhons
‫اي عملية تكون في البطن ومر عليها ( سنوات) من اهم المضاعفات الي كصير تحصل لهم هو وجود التصاقات دتخل البطن هذه‬
‫االلتصاقات تسبب انسداد في االمعاء‬

- 300-What type of mesh used in ventral wall hernia? A. Subly✅ B. Onlay (V= S ‫)ربط‬

- 301-40 years old man underwent open hernia relain and 2 weeks later presented with tenderness at
the site of hernia repair and severe parasthesia/numbness/tingling around his thigh that went down
his leg= management?
A. remove mesh staples. B. Neurectomy and mesh removal. C. NSAID✅
Key word: this case of ilioinguinal nerve inj need to NSAID for long time maybe month if no
response then nerve block and last resort is neurecromy can be done

- 302-Pt with Reducable hernia comes with intestinal obstruction manifestation and on exam was
redness and on Xray : multiple air fluid level ?
A. incarcerated (very sever pain + nusia+ vomting + herniated tissue becomes trapped and cannot
easily be moved back into place
B.strangulated(come with shock) ✅ C. obstructed(Bez say Reducable )
Key word : multiple air fluid = strangulation
- 303-60 yo pt Post ventral hernia surgery with mesh 6yrs ago pt came with abdominal distention and
on radiology cut off sign and target sign and soft tissue mass?
A. late onset crohns✅ B. adhesions with mesh C. small bowel cancer
Key word : cut of signs
- 304-t for elective ventral hernia repair, HTN and DM, on examination high JVP, LL edema, basal
crepitation, your management:
A. open surgery B. lap with mes C. delay operation until addressing current symptoms✅

- 305-Patient with femoral small hernia asymptomatic? Surgery (lap repair)


Key word: we need to distinguish between inguinal and femoral cuz belt will cause complication for
femoral so ttt is by surgery
2019 ‫ من سنة‬11 – 10 – 9 ‫تجميعات قلوري تيم لشهر‬
-
306-8 month infant with right inflamed red hemi scrotum, on examination there was red firm
irreducible painful scrotal swelling which is extended to to left inguinal region. Left testis couldn't
be palpated. What is the diagnosis?
A. Torsion appendix testis. B. testicular torsion
C. epididymo orchitis. D. incarcerated inguinal hernia✅
Key word : testicle absent

- 307-Inguinal intractable pain with hyperaesthesia with pain radiating to thigh 3 weeks post inguinal
hernia with mesh, upon physical exam, no recurrence or surgical site infection, what's the mx:
A- Nerve block B- Systemic Anti-Inflammatory✅
C- Neurectomy with mesh removal D- mesh removal
Key word: ilioinguinal nerve inj
- 308-after lap hernia repair with mesh, wound draining pus, your management:
A- explore wound + remove mesh B- put surgical drain C- antibiotic + daily dressing ✅
-
309-patient has persistent pain after hernia repair, what to do? NSAIDS then nerve block ✅

- 310-45-year-old female developed hernia located inferior and lateral to pubic tubercle. Type?
A- Femoral✅ B- Obturator. ( InFerior = Femoral ( ‫ربط‬

- 311-Patient came after 5days of hernial repair complain of inguinal mass , there is no erythema , no
tender and not associated with cough , what is most likely the cause :
A) Serosa ✅ B) Hematoma
C) wound infection ‫مافي حرارة او الم او احمرار‬ D) recurrent hernia ‫مايطلع مع الكحة‬
Key word: serosa most common complication
- 312-Surgery for hernia w/t mesh, complains of neuropain radiates to thigh, he’s already on
analgesia and once he stop it pain come back, not improve next?
A- NSAIDs. B- Nerve block. ✅ C- Remove mesh D- Remove stapes of mesh
Persistence pain after hernia repair & he’s already on analgesia and once he stop it pain come back,
not improve so we go to nerve block.

- 313-Bilateral inguinal hernia, how to manage:


A. lap repair with mesh✅ B. open repair with mesh C. lap repair without mesh
Key word: bilateral or unilateral inguinal hernia = lap repair with mesh
- 314-underwent hernia repair for right inguinal hernia, presented now complaining of that ipsilateral
testicular size is decreased after hernia repair. Dx?
A. Testicular artery occlusion. B. tight mesh C. pampiniform plexus compression✅
pampiniform plexus compression= lump in one of your testicles+ swelling in scrotum.

- 315-Lower limb paresthesia after hernia repair with mesh ?


A. nerve block ✅. B. remove mesh C. neurectomy

- 316-Elderly pt e hx ( .. months) of open hernial repair operation with mesh , complaining of groin
swelling reach scrotum +ve cough impulse how to manage ? A. Lap with mesh ✅.

- 317-Pedia no family with inguinal hernia, pedia surgeon advise treatment ?

2019 ‫ من سنة‬11 – 10 – 9 ‫تجميعات قلوري تيم لشهر‬


A.Refer to police B.Refer to social worker C.Ask ethical comity ✅ D.Refuse

- 318-60 yrs old female with femoral hernia, asymptomatic, accidental finding mx?
A.observation✅ B.open C. open with mesh D.laparoscopic
Key word: old pt with femoral hernia and asymptomatic = observation
- 319-Old man came to you with Inguinal reducible hernia, and he was advised to do hernia repair,
and PE exam was normal he is asymptomatic what will you do? no surgical treatment

- 320-Case of hiatus herni' and ask about immediate management: Lifestyle modification ✅

- 321-Making a few lifestyle changes may help control the symptoms and signs caused by
a hiatal hernia. Try to: Eat several smaller meals

- 322-Pt do hernia repair "not sure about the type" then he present with mass in the inguinal hernia
firm, regular, transpulsation "there was no Erythema or tender"and no fever :
A. saphenous B. psudoanyresm✅ C. Abscess
Key word: Pseudoaneurysm = A complication of laparoscopic inguinal hernia repair
No erythema and no tenderness

- 323-year-old weight lifter developed abdominal hernia that is irreducible, tender. Imaging
shows air-fluid levels in small bowel and no free air in large bowel. Type?
A- Irreducible. B- Incarcerated✅ C- Strangulated
**incarcerated hernia is a part of the intestine becomes trapped in the sac of a hernia—the bulge of
soft tissue that pushes through a weak spot in the abdominal wall. If part of the intestine is trapped,
stool may not be able to pass through the intestine.
**strangulated hernia occurs when the blood supply to the herniated tissue has been cut off. Pt like
Toxic hypotensive tachy ....etc.

- 324-4 y/o came to clinic w/t umbilical hernia without any Sx .., next:
A- Reassure ✅ B- Surgery C- Band
Key word: still until age of 5 year

- 325-18 year old healthy male was playing baseball and suddenly he felt abdominal pain. On
examination he has para-umbilical mass. His vital signs Bp 100/76 RR 30 HR 100 O2 sat. 95%
oxygen mask. What is your(( next)) step in management?
A.Abdominal US✅. B. CT scan. C. Erect CXR. D. Reassurance and send home
Key word: to confirm diagnosis =US
Deferential diagnosis umbilical hernia
❖ Tumor and Sarcoma :
- 326-Sclerotic lesion in distal femur:
A. osteosarcoma ✅ B. chondrosarcoma C. ewing’s sacroma
Key word: distal femur =sarcoma

2019 ‫ من سنة‬11 – 10 – 9 ‫تجميعات قلوري تيم لشهر‬


- 327-old with weight loss epigastric pain with supraclav lymph node : A. gastric cancer
Key word : signs of epigastric cancer sup LN and weight loss and if no sop LN=lymphoma
-
-
- 328-Patient w incidental finding of macroadenoma of pituitary during evaluation of
her headaches, physical and medical hx was unremarkable, what’s the best
next step? A. - ant pituitary hormone scan ✅ B. - Referral to neurosurgery

- 329-Mid-thigh lump with normal overlying skin and (+ve fluctuating) test. diagnosis?
A. Lipoma B. Sarcoma C.Aneurysm D. Sebaceous cyst✅
Key word:=sebaceous cyst low- growing ,mobile , firm, painless nodule , dark colored may be
seen on cyst , localized predominantly on the face, head, neck ,or genitals
Infection possible cause (painful, erythematous, mass exude pus )

- 330-Patient came the weight loss and abdominal distention, Ct found: Soft tissue mass retro-
peritonal , and multiply hypo-dense or hyper (forget ) liver lesions :
A- liposarcome✅ B- germ cell tumor C- lymphosarcoma
Key word: weight loss

- 331-what makes lymphadenopathy malignant more than infectious:


A. less than 5 y B. duration less than 4 weeks
C. cervical lymphadenopathy D. supraclavicular lymphadenopathy✅

- 332-Old man has progressive dysphagia e mass in mid esophagus ,chest x-ray normal . What is the
most likely cause? A. adenocarcinoman B. SSC ✅ C. Lymphoma
Key word:SCC= mild and upper
Adenocarcinoma = lower

- 333-Man with high grade dysplasia of the esophagus. How will you manage? Refer him for surgery

- 334-Smoker with chronic dyspepsia, endoscopy biopsy is squamous cell with low grade dysplasia,
what is your management?
A. PPI and rescope every 6_12 month✅
B. Refer for distal Esophageal resection
C. Observe

- 335-Most common/important risk factor for esophageal cancer is:


A. Heavy smoking✅ B. Barrett's esophagus
Key word : answer according to SSC or adenocarcinoma

2019 ‫ من سنة‬11 – 10 – 9 ‫تجميعات قلوري تيم لشهر‬


- 336-Low grade dysplasia barrett's esophagus, what is the management:
A. Pantoprazole✅ B. ranitidine ( ‫ ربط‬b= p (

- 337-what is the most common small bowel cancer ? Adenocarcinoma


‫ادينوا = دنوا = صغير‬

- 338-Epigastric pain diarrhea and peptic ulcer with + secretin ?


A. Gastrinoma ✅ B. Carcinoid

- 339-klatskin tumor= is cholangiocarcinoma located at bifurcation of common hepatic duct? CA 19-


9.

- 340-Pt known case of lung cancer and had history of (epilepsy) and on medication, medication is
contraindicantion to this pt ?
A _ nicotine replacement medication B_ Bupropion = epilepsy✅

- 341-colon cancer surgery after that he had 7ml/h urine output for 8 hours , blood pressure and heart
rate was normal what is the best management :
A. 500 NS challenge✅ B. diuretic. C. Inotropes
Key word : urine out put decrease so give normal slain
- 342-23 ylo women Typical IBS scenario her uncle in 60s has colon cancer, all blood labs normal
except mild low Hb 11 (12-15) Ask about Dx?
A. UC. B. Colon cancer✅. C. IBD

- 343-Adult patient came to a clinic with a small mass on her forearm since childhood. Asking about
the management:? A.Laser B.Excision C.follow up ✅ D.radiotherapy
‫لو تضايق المريض اشيلها أعمل ليزر‬
- 344-pt with hypertenstion and by ct abdomen show hypoechoic on adrenal gland ? biopsy
‫نتاكد هل هو خبيث او ال‬
- 345-if mass more than 6 cm do Adrenalectomy if less than 4 cm ? usully observantion
- 346-adrenal tumor with high cortisol symptoms next management :
A. Surgical resectio ✅ B. Chemotherapy C. Radiation

- 347-Old man came with vague abdominal pain with 20cm×20cm mass , finding in u/s
multiple hypoechoic masses ( or nodule ? ) What is the diagnosis :
A) libosarcoma ✅ B) lymphosarcoma
- 348-abdominal pain with 20cm×20cm mass , finding in u/s multiple hypoechoic masses?
libosarcoma
- 349-Sarcoma metastasis to ? lung
- 350-Sarcoma Biopsy most Diagnostic test:
A. Incisional biopsy ✅ B. Excisional C. Needle core
Key word: melanoma = excisional biopsy
Sarcoma = incisional
= incistion‫ساركوما = سكين = تقطيع‬
- 351-Sarcoma "no incisional in choices ? Core needle biopsy
2019 ‫ من سنة‬11 – 10 – 9 ‫تجميعات قلوري تيم لشهر‬
- 352- Most important about retroperitoneal sarcoma :
A. compress over the organs . ✅ B. Met to the near organs

- 353-Sarcom' of the thigh. Whats the best imaging for staging work up? CT ✅ sarcoma =c.t
= s=c

- 354-pt with mass 2 cm away from anal verge on proctoscopy it appears cauliflower:
A. Anal cancer B.Chondalymia acuminta ✅ ( ‫ربط‬C=c)
Key word: cauliflower
Chondalymia lata= syphilis
Chondalymia acuminate=cauliflower

-
- 355-70 year old male Weight loss fatigue proctoscopy shows mass 2 cm from anal verge
cauliflower like friable mass ?
A. anal ca ✅ B. rectal ca C. colon ca D. condylomata acu❌ (there is wight loss + mass )

- 356-Cancer in body of stomach with no metastasis and no lymph node ?


A.gastrectomy B. wide local local excision with clear margins ✅
Key word: no metastasis

❖ Chest tube:
- Hemthorax = blood accumulation in the lung ttt with chest tube
- Pneumothorax classification into:
- 1-tention pneumothorax = niddle decompression
- 2-pnemothorax = niddle decompression and then chest tube
- Symptomatic pneumothorax = niddle
- Asymptomatic = les than 2cm = observation
- More than 2 cm = niddle

- 357-by x.ray there is pneumothorax 2 cm :


A observanton ✅ B- chest tube

- 358-Pt with trauma has mild respiratory symptoms, no deviation of trachea, ...etc (all mild
symptoms) , What is the dx?
A- tension pneumothorax B- simple pneumothorax✅

2019 ‫ من سنة‬11 – 10 – 9 ‫تجميعات قلوري تيم لشهر‬


- 359-25 years male involved in RTA in ER he is fully continues and no abnormality in vital signs
admitted for observation and a FAST scan done ,.what do you expect to find?
A. Pneumothorax
B. major vessel bleeding in the thorax C. peritoneum free fluid✅
- 360-Pt came to ER after MVA with multiple trauma and hypotension, tachycardia, x ray shows
cardiomegaly and sternal fracture , Dx ? hemopericardium ✅
Key word: cardiac teponante cause hemoprtcardium (hypotension , high JVP ,muffeled heart sound
decrease)
- 361-Thoracocentesis , where to insert ( which rib) ? 9 to 10 ‫ كلمة‬Thoracocentesis ‫طويله‬

- 362-Thoracocentesis: between 9 & 10 ribs midaxillary line.

- 363-Needle thoracotomy: 2nd rib space

- 364-Elderly smoker with progressive SOB, on CXR massive Rt pleural effusion what's your next
step?
A. Thoracocentesis✅ B. Intercostal tube under water seal
Thoracocentesis .‫ نطلع السوائل ب‬plural effustion ‫عنده‬
Key word; plural effusion = thoracocentasis
- 365-lateral pleural effusion ask about site of needle insertion ? 8-10 Latral = 10

- 366-Case about pleural effusion on chest tube after a while there was blood in the water under seal
(Hemorrhagic Pleural Effusions and Hemothorax), how to manage?
A-Thoracentesis B-Tube thoracostomy ‫(فتحه‬chest tube✅ C-thoracotomy ‫قطع‬
Key word: thoracotomy if bleeding more than 2 l blood

- 367-Patient had fall 50 meter, absent lung sound on the right side, CXR was provided but not clear:
A. Intubate B. Thoracostomy✅
Key word: absent lung sound

- 368-pt fall from hight and Vitaly insatable and Pic of Cxr with symptoms and signs of
hemothorx and ask 'bout ttt :
A- Chest tube ✅ B- Thor'cotomy. C-Abx Anelgesics
chest tube‫ على طول عالجها‬hemothorx ‫معروف اول مانشوف‬

- 369-A man presented with a gun shot wound in his chest that he received an hour ago, and you’ve
done needle thoracentesis. 15 mins later 100 ml of blood is drained, what is the next most
appropriate action? A. needle thoracentesis B. Thoracostomy✅
- Key word : gun shot =thoracostomy
370-Patient who had had multiple traumas in MVA, they mention presence of free fluid in the
abdomen and spleen laceration + thoracic aortic rupture. Next ?
A. Thoracotomy✅ B. Laparotom

- 371-RTA , presented with slow breath and decrease breath sound left lower lobe lung xray picture
showed infiltrate in lower left lobe and he has lower limb fracture His blood pressure and HR was
normal ,What is your management ?

2019 ‫ من سنة‬11 – 10 – 9 ‫تجميعات قلوري تيم لشهر‬


A. thoracostomy tube✅
B. thoracotomy ‫عمليه صدر مفتوح‬
C. angiography with stent
Key word: decrease breath sound

- 372-young adult was playing football with his friend, the ball strongly hit his chest, after sometimes
he had SOB, what is the diagnosis?
A-Pneumothorax B-Pulmonary Contusion ✅ C-Tension Pneumothorax
Key word :pulmanory contusion = SOB with hypoxia ,tachypnea,tachycardia.,

- 373-RTA pt in hospital with limited facilities Found to have tension pneumothorax and fracture
femur You insert chest tube And immobilize the fracture Pt stable Decided to transfer him To
tertiary hospital , On the way to ambulance He suddenly collapsed, What is appropriate next step?
A. Intubate. B. Check chest tube insertion and functioning✅

- 374-Patient involved in RTA was transmitted to near by limited facilities hospital and the physician
decided to refer him to advanced facilities hospital Patient is unconscious:What is the most
important thing he should be maintain: intubation ✅
Key word: unconscious
- 375-left hemithorax :
A. Chest tube of left hemithorax ✅ B. Needle decompression

- 376-A known Copd patient presented with sob but was not in distress vitals stable examination was
normal.. no tracheal deviation and equal air entry on both sides..cxr shows a 2cm pneumothorax..
wat will u do ? A. Oxygen and observation B. Chest tube✅
- Key word:pneumothorax= 2cm and more chect tube less observation
- 377-Young patient comes with dyspnea and chest pain after a long flight. Upon examination:
patient is tall, thin. CXR finding: pneumothorax ,, management :
A. Thoracocentesis (with effustion)
B. Thoracostomy tube ✅
C. Conservative management
Key word : all thin =spontaneous pneumothorax
Thoracostomy inicantiom =
Pneumothorax (spontaneous, tension, iatrogenic, traumatic)
Pleural collection - Pus ( empyema), blood (hemothorax).

- 378-Patient with pneumothorax tube thoracostomy inserted after 15 min water seal bottle is filled
with blood PB: 90/60 HR: 100 Rr:22 ?
A- thoracotomy. ✅ B- thoracostomy tube
Thoracotomy is indicated when total chest tube output exceeds 1500 mL within 24 hours

- 379-Patient who was hit in the chest while playing football, after it SOB, tracheal shift ,
hypotension and raised JVP , diagnosis : Tension pneumothorax ✅
- Key word: tension pneumothorax= SOB , raised JVP, hypotension tracheal shift

❖ Intubation :

2019 ‫ من سنة‬11 – 10 – 9 ‫تجميعات قلوري تيم لشهر‬


- 380-Child with CXR finding hyperlucency of upper left zone with compression of adjacent lobe
with shift to the right, what to do?
A. thoracotomy B. Thoracostomy✅ C. Intubation

- 381-Pt with small stab wound in anterior aspect of neck. On exam he is Alert and conscious but in
labs oxygen sat 82%. What to do ?
A. Oxygen mask✅ B. Cricothyroidotomy C. Endotracheal intubation
Key word: o2 82%
- 382-Proper ET position how to chick :
A. co2 monitor✅ B. chest movement. C. easy insertion.

- 383-RTA , facial fractures , unconscious, what is opportunity in mx?


A. Cervical spine. B. airway✅

- 384-Multi trauma pt, with many facial fractures, next step?


A. secure airway ✅ B. C-collar

- 385-man with facial bones fractures after RTA, in the ER what is the first thing to do?
A. Airway✅ B. Call neurosurgeon

- 386-man after accident and resuscitation in small hospital u need to transfer to another hospital after
stabilization, it is 30 mins far. on xray u see fracture of 2-5 left ribs. no pneumothorax what will u
do?
a. intubate✅
b. call the other hospital to inform the surgeon on call
c. chest tube insertion

- 387-A scenario of a woman trapped in house that was burning and lost consciousness, Some labs
were given which I can't recall. How do you manage? intubation

- 388-Case of flail chest ask about initial thing to do? he was stable
A. IV fluid B. Assistant ventilation✅

- 389-40 something old pt came with anterior thigh stab wound 1 cm with active bleeding and
((unconscious)), most appropriate Mx (I’m not sure if it was written most next appropriate mx or
not but i think it was not written) ?
A. apply tourniquet B. Blood transfusion/ IV fluid C. intubation ✅
Key word: unconscious

2019 ‫ من سنة‬11 – 10 – 9 ‫تجميعات قلوري تيم لشهر‬


- 390-patient with trauma to face with fracture mandible resuscitation with?
A.laryngeal mask B. cricothyroidotomy ✅ C.Oral endotracheal tube
Key word : any trauma in face = cricothroidotomy

- 391-Mva with mandible fracture ? Cricothyrotomy

- 392-After an accident 18 yrs fracture of mandible and maxilla oropharyngeal tube /


tracheostomyhow we improve breathing? cricothyroidotomy

Key word: trauma in face = crico


Child: tracheo

- 393-Absolute contraindications for NG intubation include the following:


1_Severe midface trauma. ✅ 2_ Recent nasal surgery.

- 394-Chemical product burn ttt ulcer in back ?


A. wash with water for 30 mints ✅ B.intubation C.dressing

- 395-Burn on the face , GCS 15 he was alert and speaking. the two striking choosies were ?
a. Elective intubation✅ b. ICU admission for 24 hrs
key word: any burn in face = intubation
‫ربط = اغلب عمليات الوجه تكون اختيارية‬
Face burns expose patients to a higher respiratory risk, and early prophylactic intubation before
they enter the burn unit might be life-saving✅

- 396-Pt had chest trauma nad 3⁄4 rib fracture paradoxical chest movement?
‫يعني فيه كسر عظم الصدر نص مكسور ونص ال‬
•analgesia
•intubation✅ and ventilation are usually required or high concentration oxygen by mask
Key word: paradoxical chest movement =filal chest
- 397- Patient after RTA was conscious GCS 15 then suddenly collapsed u see a temporal fracture
what caused his loss of consciousness?
a. subdural hematoma b. epidural hematoma✅ c. fracture base of the skull

- 398-(( temporal fracture )))what caused his loss of consciousness? epidural hematoma ✅
Key word: temporal fracture cause epi hematoma
- 399-Young male with bilateral anterior without post lower limbs 2nd degree burn who weighs 70
kgs Asks about fluid replacement according to parkland formula:
A. 2.5L to be given in the first 8 hrs and 2.5 L in 16 hrs✅
B. 5L in first 6 and in 16 hrs
parkland form= ‫نستعمل‬
4ml x TBSA (%) x body weight (kg)
5040 ‫ تساوي‬4 ‫ نضربها بالرقم الثابت الي هو‬70 ‫ نضربها بالوزن الي هو‬18 = ‫ االثنين مع بعض‬٩ = ‫الوحدة فيهم‬lower limp ‫ال‬
٨ ‫ يعني اثنين لتر في اول‬2500 ‫ ساعة فتساوي‬١٦ ‫ ساعات والجزء الثاني في‬٨ ‫نقسمها على اثنين عشان نعطي اول جزء في اول‬
‫ساعات‬
- 400-pt with 20% back burn which indicate good response? Urine output 0.9/kg/hr

2019 ‫ من سنة‬11 – 10 – 9 ‫تجميعات قلوري تيم لشهر‬


- 401-Burn case black soot over nostrils and mouth 40% carboxyhemoglobin. (Carbon monoxide
toxicity):
A. Hyperbaric oxygen. ‫ نعطيه اوكسجين كثييير حتى يطلع‬co ✅
B. Intubation and ventilation with 100% o2.
C. Carbonic anhydrase inhibitors.
Keyword : carbon monoxide = hyperbaric
❖ Skin
- 402-Dibetic has progressive painful lump on back of neck with multiple openings :
A. cellulitis B. Abscess C. Furuncle D. carbuncle ✅ ‫مرض السكر = كرب‬
Keyword: multiple opening+ DM = carbuncle

- 403-diabetic patients with unhealed ulcer for 5 years and biopsy showed pseudoepitheliomatous
hyperplasia?
A. Biopsy B. Debridement ✅

- 404-Patient with liver mass Ct first stage filling periphery late washout centrally :
A-Hepatoma
B-hemangioma✅
C-metastatic

- 405-25yr old girl with skin lesions since childhood?


A-Laser -Excision B-Observer✅
Key word: if cosmetic = ablation
If increase = excision

- 406-Open fracture came 4 days later to ER with signs of infection ( blue, necrosis) what's the most
common organism ?
clos‫ هو‬necrosis ‫البكتيريا الي تيجي بسبب الجرح المفتوح وعدم اهتمام بنظافته وفيه عالماات‬
A-clostridium✅ B- Staph aureus C- Actinomyces israelii

- 407- case about gas gangrene due to open leg fracture infected by? Clostridium perfringens
- Key word: any opening fracture in leg cause gangrene or necrosis =c.perfri
Most common cause
- 408-Case of melanoma ? Excsion with safety margins
- Key word : melanoma dark ,irregular and ttt with excision

o Sarcoma > incisional biobsy


o Melanoma > excisional biobsy

- 409-Skin lesion irregular and dark over the knee ? excisional biopsy ✅

- 410-Old patient with 3*4 dark elevated skin lesion at the ant of his thigh, what’s the next
appropriate step? A.excision ✅ B.Punch biopsy C.Chemotherapy
- Key word: melanoma

2019 ‫ من سنة‬11 – 10 – 9 ‫تجميعات قلوري تيم لشهر‬


- 411-girl complains of rash in perianal area with pic provided (typically the same) Dx:
molluscum contagiosum
. ‫ جبت لكم كذا صورة عشان يتركز شكلها عندكم‬، ‫شكلها مثل الحبوب الصغيرة‬
.. ‫ المول سار ملوث بال‬، ‫ معناها ملوثة‬contag ‫ معناها حبيبات صغيرة و‬moll ‫ = كلمة‬molluscum = ‫ربط‬

-
- 412-ptient fall on his leg with skin loss vascular structure appear and ask 'bout ttt :
A- Debritment and 2ry skin graft ✅ B- Debritment 'nd prim'ry closure Abx
‫ معناها الجلد ماهيقدر يقفل على هذا الجرح الن جدا عكيق فالزم ناخذ‬vascular appear ‫ في الجروح ؟ اذا‬skin graft ‫متى نلتجا الى‬
...‫ونغطي فيها الجرح‬skin graft

- 413-Run over case, exposed neurovascular, most appropriate mx?


A.debridement + vacuum B. debridement + graft ✅
Key word: vascular m or tissue = second
Crash or superficial= primary
- 314-patient with post operetive 'abdominal destintion and small and large bowel obstruction
and no fever 'nd a'bdomen not tender :
A-Hypo Mg B- Hypo CL C-Hypo k ✅
= low k ‫ شلل االمعاء‬Ileus ‫كلهم صح لكن اكثر شيء يطون سبب في‬
Key word: you shod check K before and after operation the most common problem of hypoK is abd.
Distention and small large bowel obstraction
- 415-What's the cause of pain in compartment syndrome?
A• Nerve ischemia✅ B• Muscle enlargement • Bone ischemia

- 416-55 pt present to the ER after a car accident, there's a pic of CXR show aortic dissection and
lung contusion , what’s next step :
A. chest tube.
B. aortic repair and stent✅

- 417-Patient lost sensation over medial side of leg but motor function is intact. Which nerve is
injured?
A-Obturator B-Femora C-Saphenous✅ D-Sciatic
o lost sensation over = Mid thigh = obturator
o lost sensation over = medial side of leg = saphenous

❖ Diaphragmatic
- 418-Child Diaphragmatic hernia first thing you do? NGT first thing to do after stabilization

- 419-Newborn just delivered having diaphragmatic hernia , what’s the first thing to do: NGT

o Diaphragmatic Hernia
Treatment

2019 ‫ من سنة‬11 – 10 – 9 ‫تجميعات قلوري تيم لشهر‬


1_ immediate intubation required at birth: DO NOT bag mask ventilate because air will enter
stomach and further compress lungs.
2_ place large bore orogastric tube to decompress bowel.

- 420-Weber in right and bilateral + rinne test?


A-conductive hearing loss B-right schwannoma C- right presbycusis D- Left sensory ✅
Key word: weber test use in ENT
Positive rinne = normal or sensorineural
Weber shift to right so it’s either right conductive or left sensorineural

- 421-Colon cancer common metastasis to? A. Liver✅ B. Lung


‫ربط = القولون والكبد = كلهم من اجزاء الجهاز الهضمي مسؤوليين عن االكل‬

- 422-Patient complained of Rt groin pain for 2 weeks and exacerbated by exercise, physical
examination showed normal groin and genitalia, next step?
A. CT abdomen B. MRI abdomen C. Us D. Re-evaluate after 2 wks✅
‫قالك انه يزيد مع الرياضة ف غالبا حصل للعضلة شد‬
overstretching the muscles during sport.t'
‫مايحتاج اشعة‬
Key word: pt normal
- 423-Pt diagnosed with small cell lung cance, presented with dehydration, serum osmo low, urine
osmo high. ‫ هذه اعراض الجفاف‬. Ttt:
A. 5% dextrose B. Normal saline✅ C. Hypotonic saline
Key word: dehydration+ osmo low
- 424Male have problem initiating urination , bladder fell un empty after , no dribbling or ((urge
felling)) , type of incontinence ?
A. Overflow✅ B. Reflex C. Urge
Key word: non dribbling + difficult initial urination
- 425- Bradycardia (40 bpm) during laparoscopic procedure Is due to :
A. Cold gas insufflation
B. Rapid expansion of parietal peritoneum✅
C. Increase venous return
Key word: laprosc cause bradycardia = rapid expantion
- 426-60 years old patient come with sudden onset of upper abdominal pain after a few bouts of
vomiting. Examination confirme sick patient with tenderness in epigastrium and supraclavicular
subconscious emphysema . What’s Dx ?
A. esophagitis
B. acute gastritis
C. perforated peptic ulcer

2019 ‫ من سنة‬11 – 10 – 9 ‫تجميعات قلوري تيم لشهر‬


D. boerhaav’s syndrome✅(is spontaneous perforation of the esophagus )
Key word: subconscious emphysema
- 427-Pt k/c of CKD undergo apendecyomy K was 6.5 with tened T wave ? Calcium gluconate
Key word: ttt hyper K = calcium g (anti dote of K is c. gluconate)
- 428-Pt with dysuria and cloudy urine with bubbling for 2 months, hx of recurrent left iliac fossa
pain for the past 2 years ! Colonoscopy: no diverticulosis or polyp Cystoscopy: erythema on dome
of the bladder. Most likely diagnosis:
A disease Diverticular. ✅ B. sq. Cell bladder cancer C. transitional cell bladder cancer
Key word: recurrent left iliac fossa pain + cloudy =diverticular
- 429-Female with history of regurgitation and heartburn.. all conservative management failed
include PPI ..endoscopy done showed erythema and erosion suggest esophagitis?
A. manometry B. 24 h PH monitoring✅ C. Lifestyle modification

- 430-45yo female has dysphagia with liquids only retrosternal pain and regurgitation of food, what is
the test of high diagnostic value?
A. UGD. B. . Barium swallow. C. Low esophageal manometry.✅ D. CT with contrast
Key word: dysphagia to liquate = achalasia
Dysphagia to liquate and solid = GERD
- 431-Patient had a hyperextension trauma, complains of distal phalanx pain and tenderness in the
volar aspect, he also feels tenderness in the palm, what's the dx:
A- Rupture of flexor profundus✅
B- Rupture of flexor superficialis
C- intra-articular fracture of proximal phalanx
D- extra-articular fracture of distal phalanx
Memorize them as follow :
Proximal > superficial.
Distal > dee

- 432-50 male came with 4 time hematochezia , no weight loss , no abdominal pain dx:
A- diverticulosis ✅ B- internal piles C- cecum cancer

- 433-patient on TPN will increased INR what to give? vit K

- 434-Perforated appendicitis post laparotomy, presented with fever, abdominal pain CT done
showed foreign material , reexploration done, gauze was found intraabdominal ،What to do?
A-Issue complaint against assistant
B-Call your lawyer and report the incident
C-((apologize to the patient and tell her what happened ✅))

- 435-Patient with jerky carotid? Hypertrophic cardio myopathy

- 436-patient came with melena and they did upper gi endoscopy and found 1cm bleeding duodenal
ulcer and they ask about the duration of ppi ?
A. oral ppi
B. iv ppi for 24 hours then convert to oral
C. iv ppi for 72 hours then convert to oral✅

- 437-Patient post graham patch surgery, what is the most appropriate management?
2019 ‫ من سنة‬11 – 10 – 9 ‫تجميعات قلوري تيم لشهر‬
A. High dose PPI
B. IV ppi for 24 hours followed by oral PPI
C. IV ppi for 48 hours followed by oral PPI✅

- 438-6 yrs old, female presented with nausea and recurrent vomiting of every intake , pt became
lethargy with decreased skin turgor and cry with tears. Investigation revealed Normal investigations
apart from :High Na Low glucose What is the best next management :
A. isotonic saline 20ml/kg ✅ B. D10% with 1/2 saline C. oral rehydration solution

Hypovolemic Hypernatremia First correct the volume with half normal saline then once the
patient euvolemic shift to slow D5W infusion.

- 439-Old man with slow progression dysphagia, came with vomiting without blood, abd pain
associate with heartburn at eating. Abd ex normal DRE revealed slightly change stool with blood.
What is suspected diagnosis :
A. PUD. B. CA esophagus. ✅ C. CA rectum

- 440-Young female c/o fatigue and jaundice high ALP and high bilirubin Us: no any finding MRCP :
multiple stricture Next step to *confirm* Dx ?
A. Liver biopsy✅ B. Antinuclear antibody C. Repeat US D. Colonoscopy
Key word: high ALP + bilirubin multiple stricture

- 441-Case of established diagnosis of biliary sludge of postoperative sleeve pt with jaundice and US
report of biliary sludge. What is the management?
A. Expectant. B. Endoscopic sphincterotomy C. Cholecystectomy✅

- 442-What is the Pathophysiology of bacterial peritonitis due to E. Coli ?


A. translocation of bacteria
B. bowel Perforation✅

- 443-young male k/c of ulcerative colitis c/o bloody diarrhea, abd pain and S/s of intestinal
obstruction X-ray shows: megacolon What is appropriate management:
A. CT abdomen B. Ileocolectomy with. C.IV steroids✅

- 444-65 male patient present with SOB and generalized fatiguability, On Ex: non tender mass in
right iliac fossa, Hemoglobin: low What the most important step in Mx:
A. colonoscopy ‫✅اشتباه ب ورم في القولون‬
B. Percutaneous biopsy
C. Abdominal CT
D. Ultrasound

- 445-Surgeon at elective laparoscopic cholecystectomy find large mass in Stomach?


A. stop oper B. cholecystectomy only ✅ C. chlesrecomy and resection mass

- 446-Patient underwent esophageal dilation. After 12h patient had symptoms and they did
gastrografin which showed leak. Temperature is 39. BP 100/50*?
A. Stent B. Esophagectomy C. Surgical drainage✅

2019 ‫ من سنة‬11 – 10 – 9 ‫تجميعات قلوري تيم لشهر‬


(esophageal rupture treat by Tube thoracostomy (drainage with a chest tube or operative drainage
alone)

- 447-neck mass LN FNA normal thyroid cell dx : A. Follicular ca C. Papillary ca D. Ectopic✅

- 448-*Sacral ulcer, skin necrosis and exposed subcutaneous fat manage?*


A. Debridement with dressing
B. Debridement with primary closure
C. Debridement with secondary closure graft✅

- 449-Penetrating stab wound. Patient is stable. What is the next step? A. US. B. CT✅

- 450-Case of cystic fibrosis Wit abd pain , bloating , pale stool Abd scraled or curve ?
A. Chronic pancreatitis ✅ B. Acute pancreatitis C. Pseudocyst
Key word: cystic fibrosis

- 451-Patient claudication long case, then after full treatment with thrombolysis, patient is risk for
what while staying in hospital? A. Stroke B. PE✅ C. MI
- Key word : DVT case lead to p.E

- 452-Scenario Liver mass 3 cm next with splenomegaly ?


A. AFP B. triphasic CT ✅ C. Biopsy
Key word: liver mass diagnosis with triphasic CT

- 453-Epilepsy pt posterior dislocation, which joint? A. Subacromial

- 454-Tamoxifen for breast ca the came with abnormal uterine bleeding or something like that?
A. Endometrial ca ✅ B. Endometriosis

- 455-Patient has crushed by a car came with 10cm deep laceration of his thigh with continues
bleeding what you will do ?
A. tourniquet B. compress at the site of the injury✅ C. compress the femoral area

- 456-Same scenario but with out mentioning of the depth and the patient were Hypotensive ?
A. normal saline B. compress the site of the laceration ✅

- 457-Meigs syndrome How to confirm the Dx?


A. Tissue histopathology ✅
B. Tumor marker
Keyword: meigs syndrome is triad of benign (ovarian tumor +ascites + PE)
Resolved after resection of tumor
‫ = هستوا = قستوا‬megis= ‫مقاس‬

- 458-Old pt after hysterectomy what may cause her to stay more in the hospital? Her UO is 35ml per
hr ?
A. Decrease oral fluid intake✅
B. Decrease urine output

2019 ‫ من سنة‬11 – 10 – 9 ‫تجميعات قلوري تيم لشهر‬


N.B :35ml/h *24= more than 800 per days - Oligouria < 500m

- 459-Patient has hx of RTA since 2 weeks came today with SOB the chest x ray has shown
haemothorax you have put him on chest tube and want to transfer him to higher center while he was
in the ambulance the pulse oximetry drop suddenly what to do ?
A. continue the journey to the higher center
B. check the potency of the chest tube✅
C. triphasic chest intubation

- 460-Sigmoid volvulus case (s&s), ttt:


A. total colectomy with ileostomy B. sigmoidoscopy detorsion if stable✅
Keyword: children= ceacum
Elderly= sigmoid
- 461-Hepatitis B and lesion 6 cm what to do next?
A. TACE Transcatheter arterial chemoembolization) ✅more than 5 ‫حقن الكبد‬
B. Excision
C. systemic chemo therapy
Keyword: less than 5 cm do excision , more than cm do TAC
- 462-24 years old with history of appendectomy 5 years ago present with abdominal pain , distintion
, vomitting for 3 days Ct scan show obstruction signs And peritonitis , Whats contraindicated in this
patient ?
A. Propofol. B. Ketamine. C. Sevoflorane. D. Nitros Oxide✅
Keyword: obstruction c/I of nitros oxide mechanism of action of Nitero.oxide is decrease mortlity of GIT
so hncrease obstruction

- 463-patient with s&s of bowel obstructions, investigations shows air fluid level and dilated loop and
colon collapse , asking about what drug is C/I?
A. Propofol B. drug started by *sul* I think. C. nitric oxide✅

- 464A baby boy came for circumcision in the clinic. The urine was coming from the mid shaft (I
think hypospedias) what to do next?
A. open circumcision B. plastibell circumcision
C. circumcision with gomco D. refer to pediatric surgery✅

- 466-What is associated with Biliary colic?


A. Bilirubin B. Amylase C. Alkaline phosphatase. D.cholecystokinin✅
Keyword: or gall bladder stone in another Q
Cholecystokinin cause relaxation and stamulation of gall bladder, intestine ,biliary colic
This hormones for All GIT
The most common cause of gall bladder stone is biliary colic

2019 ‫ من سنة‬11 – 10 – 9 ‫تجميعات قلوري تيم لشهر‬


- 467- pt. k/c of crohn's after colon surgery developed severe diarrhea what you will give ?
A. mesalamine. B. cholystaimne✅

- 468-Patient post-surgery came for follow up develop seroma near the wound or superficial can't
remember ( no pain, no erythema, no any sign of inflammation) what your management is:
A. percutaneous drainage.
B. daily dressing✅
Keyword: pt without any signs

- 469-55 year old with constipation and distention, on examination there is , CXR pic with coffee
bean appearance, where is the site of obstruction?
A. Ceccal B. Sigmoid✅
Keyword: coffee signs

-
-
- 470-Euvolemic hyponatremia fluid replacement:
A. Normal saline B. Half normal saline
C. Hypertonic saline D. Water Restriction✅
Keyword : water restriction for loss of potassium
Euvolemic that mean increase water intake
In general , hyponatremia ttt =fluid (euvolemia)
Isotonic saline= (hypovolemia)
Hypertonic saline= (sever symptomatic hyponatremia)
- 471-Post colectomy for colon cancer she is Diabetic, received dextrose and Insulin for 2 days, then
she developed confusion and agitation. Lab : hypoNA hypoK, urine osmolality normal, serum 270.
Most likely cause?
A-Water overload.✅ B-Addison disease. C-SIADH

- 472-What of the following pass through deep inguinal ring:


A- Round ligament✅ B- illo-inguinal nerve
Keyword: round ligament in female
Spermtic cord in male

- 473-burn patient and resuscitation done, which of the following reflect a good resuscitation has
been achieved?
A-normalization of heart rate B-normalization of blood pressure
C-Urine out of 0.6ml/kg/h ✅ D-central venous pressure 12
Keyword: any burn pt give fluid
Norma urine output= in adult 30-50 ml
2019 ‫ من سنة‬11 – 10 – 9 ‫تجميعات قلوري تيم لشهر‬
In children=1ml
❖ Breast Cancer

Birad 3 =Follow up for short time. F = 3


Birard 4 = core biopsy .core ‫اربع حروف‬
Fibrocystic Changes
C/P: Diffuse nodularity that change in relation to menstraution increase in approach to menses and once
menses started it will return to the baseline
Diagnostic Modality: PE
Rx: Observation

Fibroadenoma
C/P: well-definded mass freely mobile (composed of glandular and fibrous tissue)
Diagnostic Modality: US>>core needle biopsy
Rx: Observation unless increasing in size or old age (more than40)

Fat necrosis
C/P: Hard lump following trauma or Sx.
Diagnostic Modality: Mammography
Rx: Excision

Intraductal Papilloma
C/P: unilateral breast bleeding
Diagnostic Modality: Core needle Biopsy
Rx: Excise the duct

Phyllodes Tumor:
Cystosarcoma Phyllodes
Variant of Fibroadenoma tgat has high mitotic rate classified into: benign , intermediate and malignat
C/P: large freely mobile mass +/- underlying skin changes
Diagnosis: US>> Core needle Bx
Rx:
#Smaller: Wide local excision with 1-cm -ve margin
#Larger: simple mastectomy
No Need for sentinle node biopsy

Mastitis
C/P: painfull swollen erythematous breast
Diagnostic Modality: Breast milk culture
Rx: Ab (anti staph=dicloxa or Cephalexin)

Breast Abcsess
C/P: flactuating tender palpable mass with Fever and malaise.
Diagnostic Modality: US + Culture
Rx: Driange + Ab (Antistaph) +/- ( anti MRSA=TMP/SMX Or Clinda)

Screening
Mammography=
50-74 every 2 years
Family HX at age of 40.
2019 ‫ من سنة‬11 – 10 – 9 ‫تجميعات قلوري تيم لشهر‬
Colonscopy=
every 10 years from 50 yrs.
in polyps every 3-5 years

Pap smear=
at 21-29 and every 3 years
30-65 every 3 years or 5 years if with HPV serology

DEXA= at age of 65 Yo.

Diagnostic Mammography is the 1st step in diagnosing Breast Ca. even at age of 30

Premalignant:
DCIS: From the epithelial cells of the main duct >> in postmenapousal women>>> Mammography will
show clustered microcalcification>> increasing the risk for malignancy in the same breast the same
quadrent mainly IDC(comedo type) worst prognosis
Rx:
<2cm : lumpectomy with F/U or Radiation
=>2cm: lumpectomy with 1 cm -ve margin and radiation

LCIS: arising from the cells at terminal duct lobular unit >>>> perimenapousal women(never seen in men)
>>> not palpabe or seen on mammography>> incidintal finding on Bx>>> it is a breast Cancer marker on
both Breast (bilateraly)
Rx: Bilateral Mastectomy.

Malignant Cancer
1st: IDC:
subtypes= Comedo, Colloid, sirrhous, medullary, papillary and tubular .
Mets to the LL,BB,Axillary Ln

2nd: ILC: present as ill defind thickning of the breast and it lack the microcalcification
Mets to: AMS (Axilla,Meninges and serrous)

3rd: Paget Disease of the breast


C/P: Scaly vesicular ulcerated lesion +/- bloody nipple discharge
Diagnosis: Bi-Mammo >> Wedge or punch Bx
Rx: Simple mastectomy , Breast cinserving in selected cases

4th: Inflammatory Ca:


Erythema peau d'orange and nipple retraction
Diagnosis: Skin Bx
Rx: Chemotherapy >> Sx and or radiation
- 474-Core needle biopsy showed atypical ductal hyperplasia.? wire surgical excision.
- Keyword: benign

- 475-Intraductal hyperplasia in patients with breast cancer family history? wire excision- ‫قناة نجيب لها‬
‫وير‬

2019 ‫ من سنة‬11 – 10 – 9 ‫تجميعات قلوري تيم لشهر‬


- 476-ULQ lump increasing with time, size 15x15 cm, us show solid mass with multiple cystic spaces
with posterior costic enhancement= (PHYLLOID TUMOR) = simple mastectomy

- Keyword: phylloid tumor increase with time , solid mass , not related to minstrel cycle ,painless ,

- If Q say benign do wide local excioson if malignant do simple mastectomy

- 477-radical mastectomy complaining of loss of sensation in inner side of right arm?


Inter-Costo-Bracial nerve ✅
Inter = inner Costo = mastect Bracial = arm
‫ربط= رايت =براكيال‬

- 478-lactating came with mastitis with abcsess 5 cm tx? I&D ( for abcsess)✅
keyword: any abscess do incision and drainage
mastitis is inflammation of Brest feeding (local pain and tenderness) and continue Brest feeding
- 479-Acute mastitis organism ? staph aureus= ‫ماستيتس = ماستيكا = تتعرض للستاف والهواء كثير‬

- 480-mastitis alone treat by = Antibiotic

- 481-Lactaional mastitis case = oxacillin with continu breast feeding


‫ =ربط‬O ‫ = بريست‬x

- 482-showed BIRADS IV, your next step ? Core biopsy


- keyword: birads 1.2.3.=follow up
- 4.5 = core biopsy
- 6= surgery

- 483-Breast mass 2*2 describe as Oval shape painful: breast cyst

- Keyword: brest mass in age of above 30 y

- 484-Oval with smooth surface 2*2cm not painful ? fibroadenoma

- Keyword: fibroadenoma= attack of hormone associated with manstricle cycle,painless,mobile,


smooth surfers or firm,

- 485-recent mass in right breast which is mobile ? fibroadenoma


‫ له ثالث عالمات مهمة أوال ترتبط بالهرمون وخاصة‬fibroadenoma ‫ هو ان‬fibrocyst ‫ و‬fibroadenoma ‫الفرق بين‬
‫ او قبلها ب‬menstrual cycle ‫ لذلك غالبا يجي في وقت‬، ‫باالستروجين يعني كل مازاد االستروجين يزيد حجمها وكلما قل يقل حجمه‬
‫ اذا‬cyst ‫ مافي أي الم لما تضغط عليها عكس‬painless ‫ ثاني شيء انها‬، menstrual cycle‫أسبوع وتختفي او يقل حجمها بعد‬
‫ ثابت من اسمها‬cyst ‫ الكتلة تتحرك لكن‬adenoma ‫ من اسمها‬mobile ‫ضغطت عليها تحس بالم وماله أي عالقة بالهرمونات ثالثا‬
firm or smooth‫يكون سطحها‬adenoma ‫ ال‬. ‫كيس = تابت ما تتحرك‬

- 486-no family Hx of breast ca , mammogram normal When to do next mammogram? 2year = ‫حرفين‬
mm= 2

- 487-left breast mass with bloody discharge most important next step? Bilateral Mammo
Keyword: bloody discharge and also age of women = bilateral mammo
2019 ‫ من سنة‬11 – 10 – 9 ‫تجميعات قلوري تيم لشهر‬
Us = below 30 of age and pregnant women
Mamo = above 50 every 2 years
- 488-pregnant came with bilateral breast tenderness and mass what investigation you’ll do? US ‫هنا‬
‫حامل المامو ممنوع‬

- 489-breast mass behind nipple, on US there is hypoecoich lesion, what next :


A- FNA ✅ B- core biopsy C- exicional biopsy D- reassess after ...
‫نبدء باالسهل‬

- 490-increasing breast mass 3 to 4 cm ? Bilateral ultrasound ‫حامل‬

- 491-breast mass, periareolar, nipple pulled inside, unilateral what next step:
first breast imaging, then biopsy, then staging

- 492-37 y/o Female with family hx of breast cancer complain of breast mass , She was concern of
breast cancer by ex of there was a breast mass with skin tethering Which of the following is the
most important next step ? US ✅ ‫اقل من اربعين‬

- 493-for breast examination but she wants female doctor to examine her what you will do:
A. refuse B. respect ✅

- 494-breast pain pre menses by 3 day Px normal next? reassurance.

- 495-strongest risk factor of breast ca in female : age

- 496-strong family hx of breast come with identified breast mass what next ? mamogram

- 497-young woman has painful breast lump with redness and tenderness. Vitals are given and show
T 38.9. What will u do? incision and drainage✅
Keyword: abscesses = incision
- 498-invasive intraductal papilloma. The most appropriate management is? Wide local excision.

- 499-Patient with bloody stained nipple discharge, you’re suspecting intraductal papilloma. What’s
the next step? Excision ✅ ‫نشيل فقط القناة الي فيها ورم‬

- 500-Color of discharge in intraductal papilloma ? Red


- 501-duct ectasia, color of discharge? Green

- 502-Ductal papilloma? red

- 503-breast biopsy shows intraductal papilloma what will u do ?


breast preserving✅ (Surgical excision of involved duct to ensure no atypia with breast preserving)

- 504-Management of atypical ductal hyperplasia? A. Wide local excision


‫ الي فيها سرطان واالنسجة الي حولها السليمة‬Duct ‫ فالغالج نشيل هذه‬Duct ‫قالك ورم في‬

2019 ‫ من سنة‬11 – 10 – 9 ‫تجميعات قلوري تيم لشهر‬


- 505-32 yo I think female concerned about breast cancer, because her mother have it and her sister
has ovarian cancer. What to do?nBRACA

❖ phylloid tumor

Phelloid Tx:
If he say in the case Benign > WLE
If he say in the Case milgnancy>Simple mastectomy
If he say Suspescion > You should start Mx as it Benign By WLE ‫اهم كلمة في المرض هذا لما يقولك‬
mass that has been increasing in size

- 506-Benign breast phyllodes tumor ? Wide local excision ‫فيل رجله واسعه‬

- 507-4-5 cm phyllodes mx? A.simple mastectomy B. wide local excision ✅

- Keyword: phylloid less than 10cm benign = wide locl excision

- Above 10 cm simple mastectomy

- 508-Malignancy phyllodes tumor ? Simple mastectomy ‫الن خبيث الزم استئصال‬

- 509-years lady with breast lump 2*2 i think for year suddenly the mass increased in size? phylloids

- 510-Case showed cystophylloides breast treatment? wide local excision ‫الن قال كيس‬

- 511-Lady breast mass 2 years ago but 6 month ago started to get bigger, mass well defined mobile
not attached diagnosis ? phyllode✅

- 512-with painful breast mass since 6months. It get increased.it is in the outer upper surface (around
9 clock) Histopathology confirmed a benign What is the best next step? Wide local excision

- 513-hard, mobile, well-circumscribed painless left breast mass that has been increasing in size?
Phylloid

- 514-Pt with malignant phylloid tumor what is next? A.Contrast ct of chest ✅ B.pet scan

- 516-women with 11*12 breast mass, examination showed no palpable LN. Core biopsy was taken
and showed malignant phylloid tumor, what’s the next appropriate step?
- A.WLE B.PET scan C.Chest CT witout contrast✅
- Keyword: malignant

❖ pancreatitis
- 517-typical pancreatitis: epigastric pain, high amylase what is next step? A. US ✅ B. CT scan
- Keyword: in pancreatitis :best initial = Us
- Confirm= ct
- Amylase = sensitive

2019 ‫ من سنة‬11 – 10 – 9 ‫تجميعات قلوري تيم لشهر‬


- Lipase = specific

- 518-high amylase + k/c of gallstone what you will do next? A. US


Keyword: us is the best for gallstone
- 519- signs and symptoms of cholangitis of biliary pancreatitis, q was "What is the most important
initial mx"? IV fluids

- 520-Scenario of abd pain post meal with a hx of pancreatitis month ago Fluid collection found what
is it? Pseudo.Cyst
Keyword; complication of pancreatitis is pseudo. cyct

- 521-Pancreatitis 5 weeks ago. Now she has epigastric tenderness and cannot tolerate food with
vomiting each time. By ultrasound you found large about 12X10 mass With thick wall and fluid
inside. Labs: 346 amylase, Wbc 15k. What is the diagnosis?
A. Pseudocyst B. Abscess C. Walled off necrosis✅

- 522- chronic pancreatitis what you will find? increase lipolysis


- Keyword: chronic pancrititis will found increase of lipolysis and hyperglaicemia and hypocalcemia
-

- 523-Chronic pancreatitis what you will find? hyperglycemia, hypocalcemia+increase lipolysis

- 524-40 years old woman with no pain but you noticed jaundice. She has high direct bilirubin and
high ALT what is your diagnosis? Carcinoma in head of pancreas ✅

- Keyword: painless jundice

- 525-Pt with pulsatile epigastric mass and ask about investigation?


- A-Serum k,
- B-Serum amylase ✅
Keyword: this case aortic aneurysm so to roll out pancreatic do serum amylase if high = pancreatic

Pt with AAA clinical finding most appropriate next alboratoryinvestigation:


A-serum amylase✅
b-calcium
- 526-Diffuse guarding and sluggish bowl sound amylase was hight what is the next step:
a-erect chest x-ray✅. b- abdominal x-ray CT abdomen
‫صوت االمعاء نسمعها بالصدر معناها حاصل ثقب‬
Keyword:sluggish
- 527-case of acute pancreatities ?Iv fluids and analgesic Abx for necrosis
- Keyword: acut pancreatic start with Npo and then iv fluid then analgesic
2019 ‫ من سنة‬11 – 10 – 9 ‫تجميعات قلوري تيم لشهر‬
- 528-Diagnosed case of acute pancreatitis. Received IVF + analgesic Next step?
Urgent surgical consult'tion✅✅

- 529-30 y/o Cystic fibrosis with GI Symptoms what complication he might have?
A- Acute pancreatitis B-Chronic Pancreatitis✅
Keyword: cystic fibrosis= chronic pancreatic
❖ pancreatic pseudo cyst
Most pseudocysts resolve without interference and only require supportive care. For some, drainage
is indicated.

- 530-best method for pancreatic pseudocyst drain ? Endoscopic


- Keyword: pseudo cyct drain if no infective= endoscopic
- If infective(fever , leukocytosis ) =precetsnous drainge

- 531-Treatment of *Infected* pancreatic pseudo cyst: Percutaneous drainage ✅


endoscopic‫ وهي‬cyst ‫ غير كذا دايم اختاروا افضل طريقة نشيل فيها‬percutneus ‫ اختاروا‬cyst ‫اذا قالك انفكشن‬

- 532-Patient recovered from acute pancreatitis episode , presents 6weeks after with vomiting and
epigastric fullness CT done:showed cystic collection behind the pancreas what is the most likely
dx: (no fever)
A-Pseudo cyst ✅ B-Pancreatic abscess C-Pancreatic necrosis

- 533-pancreatic pseudocyst, one is 18 cm x 24 cm , how to manage ?


A- per cutaneous aspiration B- endoscopic drainage ✅

Pancreatic pseudocyst ttt:


- less than 6 cm and 6 week > observation
- more than 6 cm and 6 week > drainage

* Percutaneous catheter drainage for (( *infective* pseudocysts )) .


* Endoscopic drainage = ‫ ما عدا انفيكتف‬، ‫ هذا عادي نستعملها مع كل المرضى‬.

- 534-Pancreatic pseudocyst 5 week and 18 cm ? A. Medical B. Surgical✅ ‫ سم‬٦ ‫اكثر من‬


- Keyword: more than 6cm =drainage
- Less than 6 cm =observe

- 535-Pancreatic pseudocyst if there is sign of infection as fever and leukocytosis do ? Percutaneous


drainage

- 536-Alcoholic pt has severe abd pain radiate to back ?


A- stomach cancer B_ alcoholic pancreatitis✅

- 537-Gray turner sign = its sign of ? Necrotizin pancreatitis = causes of =abdominal hemorrhage
(bleeding) ‫جراي = اجري عشان تقوي عضالت بطنك‬

- 547-Poor prognosis of pancreatitis:


A. Hematocrit ✅ B- Amylase above something

2019 ‫ من سنة‬11 – 10 – 9 ‫تجميعات قلوري تيم لشهر‬


‫هي مات = مات = عالمة سيئة‬
Keyword: poor prognosis of pancritis have lo ca hematocrit

- 548-Epigastric pain radiate to the back x Ray normal, high amylase, ct showed pancreatic swelling,
what is this complication? pancreatic pseudocyst

- 549-Old with painless jaundice . Us show dilated gall bladder,intra and extra hepatic bile duct . Ask
about dx :- A. Klutskin Tumor (shrinking gall bladder) B.Pancreas Adenocarcinoma✅
Keyword: extra hepatic and gall bladder dilated

- 550-Pancreatitis first first step? IVF

- 551-Case of pancreatitis Admitted, given ivf and pain killer Next to add in managment?
A.Ppi B. Antibiotics C.Urgent surgical consultation✅

- 552-13X15 pseudo cyst of the pancreas in the lesser sac Patient vitals showed high temperature
treatment: A. endoscopic. B. per cutaneous✅ ‫الن انفكشن‬

- 553-Pt. With pictures of pancreatitis ( epigastric pain + high amylase) hxof gall bladder stone ,Next
important initial step ? A. Crystalloid fluid iv ✅. B. ERCP. (when pt stable) C. US
- Keyword: initial = iv fluid then US
- ERCP=is urgent only when its cholangitis

- 554-Pancreatic pseudocyst classical presentation. No signs of infection, there was 15×16 cm fluid
collection in the lesser sac. What is the management?
A. Imaging guided aspiration B. Surgical drainage C. Cyst excision D. Endoscopic fluid aspiration.✅

- 555- 50 year old man presents with progressive jaundice, dark urine, and right upper quadrant pain
and distention. On physical examination he has a palpable gall bladder. Imaging shows an enlarged
gall bladder and dilated Intrahepatic duct. Amylase = 481. diagnosis is?
A. klatskin tumorn B. Pancreatic cancer ✅ C. Cholecystitis D. Mirrizi’s syndrome

- 556-complication of pancreatitis after conservative treatment was done: Pancreatic pseudocyst

- 557-pt had pancreatitis and gall stone , had been managed with fluid and stable what to do :
A. Cholecystectomy at the same admission ✅ B. Cholecystectomy after 4-6 weeks
N.B: Acute biliary pancreatitis: First resuscitation specially IVF.
Then : ERCP (stone extraction) followed by lap chole. Abx not indicated unless severe necrotizing
pancreatitis. If it was mild, moderate: lap chole in same admission.
Severe: lap chole in 4-6 weeks.

❖ Carpal tunnel syndrome



❖ carpal tunnel syndrome is peripheral neuropathy (numbness- tingles- ) associated with pregnant
and DM –hypothyroidism
❖ Ttt: best entail therapy(rest splint to relive the pressure – physical therapy-surgical)
2019 ‫ من سنة‬11 – 10 – 9 ‫تجميعات قلوري تيم لشهر‬
❖ After surgery the pt give anisette
- 558-Carpal Pain directly at the incision is typically only present for days or weeks after the surgery.
Protecting the incision can help alleviate pain, and it's important to avoid lifting or gripping for
several weeks after carpal tunnel surgery. ... Treatments for pillar pain may include rest, massage,
and hand therapy
- 559-Carpal tunnel syndrome still in pain after surgery?
A- Physiotherapy✅. B- Analgesia. C- Open again
‫ هو انه االلم يستمر بعد العملية وممكن كمان اشد من االول‬...... pillar pain carbal tunnel =
- 560-Patient has nerve radial injury , where level of injury ? spiral groove humerus
...............‫ = راديال ريدي = هل انت ريدي مستعد للهجووم ؟‬radial = ‫ = هيوميرال = هجوووم‬humeral = ‫ربط‬

- 561-Patient had loss of sensation on the snuff box and dorsum of the medial hand, hehad wrist drop.
At which level is the radial nerve injured?
A. Axilla. B. Humerus groove✅ C. Olecranon D. Carpal tunnel.
Snuff box> radial> spiral groove of humerus
humeral = humen
keyword: wrist drop = radial N inj. Or 3 and half finger with weak sensation

- 562-Paraesthesia and numbness with wrist drop (shows radial nerve injury) at which Part ?
A. Groove of the humerus ✅. B. Carpal tunnel. C. Olecranon
Wrist = Radial.

- 563-Typist c/o tingling and pis in left thumb, index and middle & problem in dorsiflexion and
fingers extending which nerve affected ? A. Median. B. Radial ✅

- 564-Patient with pain when writing at keyboard, some test showed hypoperfusion to superficial
palmar arch, what’s the artery affected?‫ يسال عن الشريان‬ulnar .

- 565-Thenar muscle atrophy ? Median nerve

- Keyword: right and hypothenar= ulner

-566-Female complaining of tingling sensation in her ring finger increase when she raise her hand ,
in the examination positive arm elevation test? carpel tunnel syndrome ✅
Keyword: tingling + ring finger
- 567-Pt do surgery then Loss of sensation in ear pinna and upper neck what nerve is injured ? Great
auricular nerve
:)‫ عشان المجوهرات‬pinna ‫ جزء الكبير والمهم في االذن هو‬great ‫ و‬، ‫ = معناها اذن‬aurical ‫ربط‬

- 568-Numbness on thumb and index finger what is the nerve affected? Median nerve

- 569-Male with left little finger and(( ring numbness)) or pain , raisstress test increases the
symptoms, what’s dx: thoracic plexus

- Keyword :little finger affected when raise his hand above and feeling numbness so thoracic plexus
is block = thoracic outlet syndrome
2019 ‫ من سنة‬11 – 10 – 9 ‫تجميعات قلوري تيم لشهر‬
- 570-Athlete with pain during standing PE: tenderness in medline planter surface? plantar fasciitis.

❖ Gallbladder
- 571- Most common cause of biliary colic ? A. cholydocolithaiasisis B. Gallstone✅

- 572-An elderly with IHD day 2 post cholecystectomy presented with sudden chest pain SOB and
vitally hypotension and tachycardia whats best ?
A. . CXR. B. ECG. C. CT angio✅ D. LL duplex US
Keyword: SOB+ hypotension +tachycardia =PE
- 573-Cause of biliary colic?
A. Choledocholithiasis B. Gallbladder stone C. Gallbladder sludge✅
Keyword: gallbladder sludge is a collection of cholesterol, calcium, bilirubin, and other compounds
the build up in the gallbladder, it is something's called biliary sludge because it occurs when bile stay
in the gallbladder for too long , bile is a greenish-yellow fluid that product in the liver and stored in
the gallbladder
- 574- pt with typical cholecystitis Mx? A. immediate lap chole ✅ B. wait
- Keyword: if pt old= conservative mange
- If young= immediate

- 576-what’s the common cause of cholesterol gallbladder stone : A. obesity B. rapid weight
loss✅

- Keyword: rapid weight loss cause mixed stone

- Acute cholecystits =biliary colic+ infection

- 577-Colicy abd pain with dilate CBD and intra hepatic duct causes of pain? Constructions of
Sphincter of oddi

- 578-Abdominal pain and US show stones in gall bladder with(( normal wall ))what is the
management ?
a) Lap chole b) Ursodeoxycholic ✅
Ursodeoxycholic Acid, is a naturally occurring bile acid and is used to dissolve gallstones that are
rich in cholesterol. It is also used to improve the flow of bile in primary biliary cirrhosis.

- 579-Old man came with jaundice and dark urine , palpable gallbladder , lab shows direct bilirubin?
A) klatskin tumor. ✅ B) Perihailar tumor. C) CBD stone
Keyword: old + palpable gallbladder+ pain less jaundice= klatkin tumor
- 578-30 y/o male patient presented with abdominal pain related to meals, radiated to the back, labs
resulted high amylase, U/S showed bile sludge with no gallbladder stones were notified, no CBD
dilatation, what’s your management?
A.Labroscopic cholycystectomy . ✅ B. Endoscopic U/S C. Endoscopic sphenoidotomy
Keyword: this case of cholecystitis present with fatty meal
Ttt of cholecystitis = urgen

2019 ‫ من سنة‬11 – 10 – 9 ‫تجميعات قلوري تيم لشهر‬


Choleithiasis = elective

- 579-Patient during complicated lap chole the surgeon accidentally transected the CBD (common
bile duct( above the level of cystic duct, what’s your management?
A.hepaticoduodenostomy. B. hepaticojejunostomy ✅ C. choledocho- duodenostomy.

- 589-Lowest risk for cholesterol gallstones ? Nulliparity ✅

- 599-pt. With RUQ pain, US showed fluid around gallbladder, management:


A- US guided aspiration ✅ B- lap choly
Keyword: RUQ pain and US showed fluid around gallbladder = aspiration
- Recurrent RUQ pain aggregation with morphine, temperature 36.5 ?
a) Biliary colic ✅ ‫ نستخدم بديل له ميبرادين‬، ‫دواء المورفين يزود جدا االلم وال يخففه‬

- 600-Pt came to ER with ruq pain and jaundice, with fever i think, 2 h later patient improved, us
showed multiple gallstones within normal walled gallbladder, management:
A. lap choly✅ B.follow up C.ursodeoxycholic

- 601-Pt post PTC has fever, on PR exam ant. Boggy mx ? A. Percut. Drainage✅ B. IV Abx (if
small)
- Keyword: boggy that mean abscess

- 602-RUQ pain on US Dilated common bile duct with intrahepatic biliary dilatation , what is the
cause ?
cholelithiasis (gallstone)
keyword: intrahepatic dilatation
- 603-Patient with jaundice, abd pain and on US u see stones and dilated CBD vitals show fever only
asking about diagnosis?
a. ascending cholangitis✅ b. choledocholithiasis c. cholecystitis
Cholangitis is an inflammation of the bile duct system ( dilated CBD so inflammantion on bile duct).
Keyword: fever+ jaundice +abd pain=ascending cholangitis
- 604-Pt with DM and HTN and SCA , type of gallbladder stones will form?
A. Mixed B. Cholesterol stone C. Pigmented >> IF SICKLER✅
Keyword: SCD= pigmented
SC trait= mixed

- 605-Diabetic obese elderly female with sickle cell trait has Lithiasis what kind of stone she has?
Mixed ‫ترايات = ثالثة = ميكس = كذا رقم‬

- 606-Patient with history of lap chole few weeks ago presenting with right upper abdominal pain ,
respiratory symptoms , US done and show pus collection at site of gallbladder 12*6 cm What is the
most appropriate in management?
A. antibiotic B. precautions guide drainage✅

- 607-Patient post bariatric surgery complains of on and off fever for one week On examination chest,
abdomen and wound were normal How are you going tomanage:

2019 ‫ من سنة‬11 – 10 – 9 ‫تجميعات قلوري تيم لشهر‬


A. Reassure B. CT abdomen✅ C. Chest x-ray
Keyword: : Spiking fever > deep collections

- 608-female pt present with right upper Q pain , )))febrile((( Lab test show: increased WBCs
increase ALP increase Direct bilirubin US Show : fluid around gallbladder , multiple stone What the
most appropriate management ?
A. Lap chole B. laparotomy C. precautions guide dringe D. antibiotic✅
Keyword: fever + increase Direct bilirubin (more than 50%) = ostrective jundicce

- 609-Elderly pt admitted to Icu with acute MI , and developed pneumonia on tazocin , he also have
RUQ pain and tenderness. Management ?
A. ercp drainage B. emergency cholecystectomy
C. convert from tazocin to meropenem D. us guided cholecystostomy drainage✅

- 610-post cholecystectomy came with perihepatic collection, what will you do?
Percutanous driange✅

- 611-patient with diverticultits did sigemoidectomy , 5 or 7 days after had fever on per rectal exam
there is bogginess anteriorly what to do : A-drainage ✅ b-reassure
- Keyword: bogginess

- 612-Post colectomy paracolic collection:management?


A-Ultrasound guided drainage ✅ B-open drainage

- 613-ulcerative colitis pt with y shape something with very enlarged transverse colon and no
haustrea what is Rx ?
A.proctocolectomy with ileal pouch B. pan colectomy with ileostomy✅
Keyword: Y shape if didn’t =observed with NPO iv and steroid

❖ ERCP
- 614-Obstrictive jaundice pic and cholangitis on IV abx ,US (dilated ducts, gallbladder has
stones) what else in the mx? A-ERCP✅ B-cholecystectomy
Keyword: cholangitis ttt with ERCP
- 615-Typical pic of acute cholangitis, next investigation? A. US B. MRCP C. ERCP✅
Cholangitis is an inflammation of the bile duct system it happened bez there is stone on bile duct and any
stone on bile duct best manag with ERCP
common bile duct ‫ مثل ما يقولكم‬dialated ‫ سواء كانت‬bile duct ‫ او جابوا سيرة‬bile duct ‫ في‬stone ‫أي مرة نشوف‬
ERCP‫اختاروا دائما‬dialated
Keyword: (cholangitis) ERCP is =confirm
MRCP = diagnostic
US= initial and first thing to do

- 616-Pt with fever, jaundice and RUQ pain,initial management? A. abd US B. ERCP✅ C.
MRCP
- 617-Pt with history of Cholelithiasis, today present with abdominal pain after fatty meal, on US :
multiple gallbladder stones, thick wall , ))CBD 12(( mm , what’s next: A-ERCP
- B-cholecystectomy ✅
2019 ‫ من سنة‬11 – 10 – 9 ‫تجميعات قلوري تيم لشهر‬
- Keyword: cholelithiasis is gallbladder stone cholestrole stone 10%and pigmented stone 20%
- Investigation: : US is the best
- Cholangitis inflmation in biliary duct(with fever) and choleothiasis is ston in gall bladder

- 618-Pt with calculous cholecystitis come with acute attack, When to do operation to Her ?
A. Lapcholy as soon as possible ✅ B. Wait 2 or 3months
Keyword: inflammation so we urgent within 72 h
When the diagnosis of calculus cholecystitis is established immediate intervention is indicated because of
the high risk of rapid deterioration and gallbladder perforation

- 619-patient presented with signs and symptoms of cholecystitis, days after hospitalization the
patient recovered clinically and her labs returned back to normal except for AMYLASE, what is
your management: ‫ معناها حصل التهب في البنكرياس بسبب حصوة‬ERCP ✅.
Keyword: is stone cause inflammation of pancreatic and amylase high so do ERCP for diagnosis and
therapeutic
- 620-Post lap chole presented after few days with abd distension and ascites and abd pain what will u
do?
a. open b. lap c. tapping D . ERCP✅
keyword:

- 621-After long ERCP, patient hypotensive with pain ..Most common site injured:
A. Esophagus B. Duodenal ✅ C. Gastric

- 622-Patient post cholecystectomy day 8or 9 develops right mouth corner pain and fever 38.5
what is your mx: A-Paracetamol. B- Antibiotic✅.
- Keyword: infection this is acute peritonitis post surgery due to ascending of Bactria during
intubation
- This is acute supportive parotitis post surgery due to ascending of bactria during intubation
,ABand analgesia

- 623-Patient came with RUQ pain , ultrasound findings : thickining gallbladder wall , pericystic fluid
and stones . What's best intervention :
A) percutaneous drainage B) Laparoscopic cholecystectomy ✅

- 624-6 weeks Post MI patient, planned to do cholecystectomy:


A. Do Lap cholecystectomy now ✅
B. Do Open cholecystectomy now
C. Do lap cholecystectomy after 6 months

Gallbladder polyp *less than 10cm* :-


*if elderly or with stone* > cholecystectomy
*if less than 50y >* follow up with us every 6m

- 625-Pt did abdominal US as htn screen and the results show gallstone polyp what to do?
.Follow up in 6 month ✅
Keyword: less than 1 cm = follow up , more than 1cm =cholecystectomy

2019 ‫ من سنة‬11 – 10 – 9 ‫تجميعات قلوري تيم لشهر‬


- 626- Patient with biliary polyp, 0.6cm. Management: A. reassure. B. follow up✅

- 627-Cholecystectomy 6 years back, presented with vomiting and increased bowel sound movement
(exaggerated) ? A. Adhesion ✅

- 628-Kc of small gallstones presented with jaundice, Labs show high alk phos, high cong bilirubin,
Next? A. Ercp B. Abd us ✅✅
Keyword: gallstone You have to start by US first If revealed dilated CBD go for ERCP

Acute cholecystitis presents with right upper quadrant pain, fever, and leukocytosis. Patients with
acute cholecystitis should be treated with laparoscopic cholecystectomy within 72 hours

- 629-30y female patient, came to ER with abdominal pain... She has hx of elective cholecystectomy
due to Gallstones. Now she has high amylase, 9mm CBD and jaundiced diagnosis? missed stone in
CBD
- 630-Lady post cholecystectomy , due to cholothiasis. week later presented with jaundice , pain ,
U/S: showed dilated CBD. Dx: Retained stones
- Keyword: post chlecystomy jaundice is missed or retained stone or reccurent stone
- Presentation: if present 2 year after operation = recurrent
- If present before 2 year after operation = missed
- 631-70 years old patient with a history of Myocardial infarct that was 6 months ago, on this
presentation he has cholecystitis and requires cholecystectomy. when will you operate?
A. now✅ B. After 6 months

- 632-the surgeon accidentally cut off the Common bile duct at a level that is just above the cystic
duct, how to repair? hepaticojejunostomy

- 633-laparoscopic cholecystectomy come with discharge from middle management? ‫يعني الجرح ينزل‬
‫ منه صديد‬A.daily dressing B- wound inspect. C- exploration✅

- 634-Middle discharge after cholecystectomy? A. Abs B. Exploration✅(No dressing or


inspection)

- 635-Patient on 3rd day post cholecystectomy develops fever, has no abdominal pain.What is the
likely cause of fever: A.UTI✅ B.Wound infection C.Chest infection
3 day = UTI = ‫ثالث حروف‬

- 636-US showed collection 10x15cm in gall bladder fossa, ttt:


A. physiotherapy B. iv abx C. drainage✅

- 637-PatiePT WITH post cholecystectomy, presented early (don't mention which day) with fever and
lung consolidation (atelectasis), and US showed 10*12 collection of fluid in lesser sac ,Mx?
A. Physiotherapy B. abx C. Precut drainage✅
Keyword: abscess

2019 ‫ من سنة‬11 – 10 – 9 ‫تجميعات قلوري تيم لشهر‬


- 638-klatskin tumor = is cholangiocarcinoma located at bifurcation of common hepatic duct ? CA
19-9.

- 639-RUQ pain and dilated common bile duct, what is the diagnosis? Choledocholithiasis ✅
Common bile duct stone, also known as choledocholithiasis, is the presence of gallstones in the common
bile duct (CBD) (thus choledocho- + lithiasis)

- 640-Choliduocholithisis managment? ERCP

❖ GIT
- 641-Pt alcoholic ,smoker came with sever pain generalize in the abdomen In examination was
tenderness and guarding? Duodenal perforation
Keyword: anther Q
Patient with sudden epigastria pain radiated to back, X-ray show air under diaphragm what is the most
likely diagnosis? A- perforated duodenal ulcer
b- acute pancreatic
- 642-Elderly smoker with dysphagia , showed high grade dysplasia , management?
A. Add ranitidine B. Ask him to stop smoking C. congenital behavior therapy D_ endoscopic
mucosal resection ✅
Keyword: smoker+ GERD +high grade dysplasia = esophagectomy (esophageal ca)

- 643-best diagnostic test to detect ischemia : stress Echo

- 644-sign in radiology for duodenal ulcer? clover leaf


= love = clove‫ = ودود‬doudenum = ‫ربط‬

- 645-Old man suddenly had vomited two episodes of bloody vomit, Physical Exam shows no
Tenderness over his abdomen or sternal area, he is stable, what is your diagnosis: Mallory weiss tear

- 646-24 hours vomiting after food poisoning develop hematemesis on endoscopy blood streak on
stomach Dx? Mallory Weiss tear

- Keyword: Mallory is upper epigastria bleeding after prolong vomiting (painless repeated
vomiting+old)

- 647-adult patient ( i forgot the age may be in 30 ) have repetitive vomiting come with mild bleeding
( mallory weiss syndrome )what is the appropriate management?
A. conservative management ✅ B. laparotomy exploratory C. laparotomy with resection
Keyword: Mallory Weiss ttt with conservative mange and in sever case ttt injection epinfrine

- 648-C/I to liver transplantation? Cirrhosis with active alcohol✅


Keyword: any pt with alcoholic and acute hepatitis Ci of transplantation
- 649-patient with hepatitis B cirrhosis with ascites and have 6 cm lesion found in the with high
vascularity = treatment? A. anti viral. B. resection. C. chemo embolization✅
- Keyword: c,never resect in cirrhosis

- 650-pt with Chronic hepatitis b then , then discover 70% of the liver was multiple lesions, next:
Colonoscopy.
2019 ‫ من سنة‬11 – 10 – 9 ‫تجميعات قلوري تيم لشهر‬
Keyword: colorectal cancer metastasis to liver
- 651-Male Pt, smoker, have a history of appendectomy, his brother have crohn’s , what’s the risk
factor to develop crohn’s : A. male B. smoking C. family history ✅ D. history of
appendectomy

- 652-Healthy female with dysphagia diagnosed as achalasia, best treatment ?


A- pneumatic dilatation ✅ B- botulinum injection C- fundoplication
Keyword: anther Q
Female pt present with achalasia what is the most appropriate mangment?
Pmeumatic dilatation
Botox inj
Funoplcation
Myotomy
Achalasia = initial therapy is pneumatic dilatation
Boyulinum toxin is second line use in pt who are poor surgical cadidates
CCB third line
- 653-Valvulas location: A.Cecum B.rectum C. sigmoid ✅

- 654-Y shaped colon, Sigmoid volvulus what to do? Colonoscopy or sigmoidoscopy


Keyword : ttt volvulus : first sigmoidoscopy then colonoscopy then CT
- 655-Old patient history of vomiting with palpable mass imaging finding Y shape next app action?
A. Fleet enema B. Ct C. Sigmoidoscopy D. Colonoscopy
Keyword:
- 657-Old male with generalized abdominal pain, ascites, constipation X-ray >> massively enlarge
loop up to right part ? A. Sigmoid volvulus ✅ B. Acute diverticulitis
Keyword:
- 658-Patient came with abdominal diatention x ray showed y shaped colon what’s your
management?
A. Fleet enema B. Sigmoidectomy with end colostomy C. colonoscopy✅(obstruction)

- 659- Pt with UC the most associated risk of Cancer is? primary sclerosing cholangitis✅

- 660-Case of Sclerosing cholangitis, what you do for the patient? A- CT B- Colonoscopy ✅

- 661-Abdominal distended, vomiting, picture of obstruction. Dx? volvulus


Keyword: volvulus accore end old age and infant
- 662-Management of unstable patient with sigmoid volvulus? Sigmoidectomy
Keyword: first ttt stabilization then sogmdectomy
663- Most common site of sigmoid volvulus? ➔ Elderly > sigmoid ➔ Children > Ceacum

- 664-Patient with basal ganglia disorder and absent gag reflex. How to provide nutrition?
A-Gastrostomy ✅ B-Jejunostomy C-NGT

665-Colorectal surgeon performing a low anterior resection for CRC and the pelvis won’t stop bleeding, so
he consults a vascular surgeon and he does:
A-Heavy packing of pelvis ✅ B-Arteriography intra operatively C-Infraceliac clamp
Keyword: bleeding in vascular
- 667-bleeding aortic in OR and you did packing but he is still bleeding. Next?
2019 ‫ من سنة‬11 – 10 – 9 ‫تجميعات قلوري تيم لشهر‬
A. clamping infrarenal✅ B. clamping supraceliac C. cable CTA
Keyword: we have 2 artery that blood supply to body inf. Vina cava and descending aorta so we do
clamping to artery to stop bleeding
Packing isn’t enough and bleeding is high = sup celiac clamp
Packing enough and bleeding if pelvic = infrarenal clamp
- 668-Patient had RTA with seat belt sign x ray of the spine showed chance fracture what most
probably you will find in ? Duodenal perforation
- 669-RTA patient his speed was 130 and he was on seat belt came with stable status no hypotension
or change in consciousness what you will do ? A. CT ✅ B. us of abdomen. C. laparotomy

- 670-Pt elderly known to have IHD come with sever central abd.pain Amylase: slightly elevated =
Mesentric emboli of occlusion ‫اي مرة تشوفوا مكتوب حصل له اسكيميا قبل كذا بالقلب وبعدها سار له الم بطنه اعرف‬
‫انه جلطة حصلت ايضا باالمعاء خااااصة كبار بالعمر‬
Keyword: ischemic heart disease (sever pain- vomiting –abd distention – cardiac disease) cause by
mesenteric occlusive ischemia
- 671-Patient known to have ischemic heart disease complain of abdominal pain (the only complain)
with lab showing amylase of 600 and x-ray

- 672- showing signs of obstruction ?


A.Acute appendicitis B. intestinal obstruction C. mesentric ischemia ✅

- 673-55 years old male presented with sudden abdominal pain radi ated to the back the patient has
history of cardiomyopathy, the patient mentioned that he passed small amount of loose stool, during
the examination there is exaggerated bowel sound Labs: Amylase = mildly elevated above normal
What is the Diagnosis? A.Acute pancreatitis B. Mesenteric vascular ischemia✅

- 674-seat belt sign bowel/mesenteric injury like Duodenal perforation.


Keyword: seat belt signs came with small intestinal inj– mesenteric inj- abd content inj
- 675-multiple ulcers in the antrum ? antrectomy ##heavy

- 676-Multiple antrum ulcers , by Bx >H pylori what is the mx ?


A. start Abx ✅ B. Chemo C. Radiation
Keyword: H. pylori cause ulcer
- 677-GIST , 5 cm in posterior wall, mx: A-Total gastrectomy. B. partial gastrectomy. C. WLE✅
Keyword: GIST (in gastro intestinal tract or stomach or small intestine )less than 2 cm= observation
More than 2cm=wide local excision
Treatment of GIST
Small less than 2 cm = observation
Large more than 2 cm =surgical excision + adjuvant
adjuvant = after surgery give chemo – now adjuvant =give chemo before and after surgery

- 678-gastrointestinal stromal tumors with ((metastasis)) = treat by ? tyrosine kinase inhibitors


(imatinib)

- 679-Giant gastrointestinal stromal tumor = wedge wide local exsion ? Dont choice (gastrectomy)

- 680-stomach tumor with positive secretin stimulation test ? gastrinoma

2019 ‫ من سنة‬11 – 10 – 9 ‫تجميعات قلوري تيم لشهر‬


- Keyword: zollinger – Ellison syndrome

- 681-pt with LLQ pain, and hx of constipation, on exam, bulky mass in LLQ, with wild discomfort,
no worrying sings or old age, dx: constipation ✅
Keyword: constipation for 3 w
682-50 yrs man diabetic, well controlled had colon cancer surgery ( coloectomy),, they kept him on
insulin and dextrose, after surgery by 2 days he became irritable, in shock, his electrolyte ( Na 129 )
( K 3.2 ) urine and serum osmolality Normal, what's the Dx: Fluid overload ✅

- 683-Patient while doing laparoscopy for esophageal perforation with Bp 80/50,,RR25 pulse 120
which type of shock? A.Cardiogenic B.Hypovolemic ✅‫نزيف بسبب الثقب‬
Keyword: perforation = bleeding
- 684-patient with GERD underwent endoscopy show multiple antral ulcer Ttt?
Partial distal gastrectomy ✅ ‫= هي نفسها‬antrectomy

- 685-Succession splash -‫ هذا اسم صوت بالسيتيسكوب نسمعه في البطن عن الفحص‬symptoms of obstruction
patient have? A.met acidosis B.Met alkalosis ✅
= alk ...........‫ =صوت = قلق‬succ = ‫ربط‬
Keyword: vomiting
- 686-Symptoms of obstruction and X-ray done showed Dilated loops towards RUQ ? Sigmoid
megacolon
Keyword: dilatation to loops= sigmoid megacolon
- 687-K/c of chrons with perianal mass , painful , there discharge when touch it , Mx? MRI pelvic

(Angiodysplasia is a small vascular malformation of the gut. It causes sever bleeding therfore must be
treat )
*Treatment for angiodysplasia*

- 688-Angiodysplasia in 60 year patient, how to manage? argon plasma coagulation (APC)‫نكوي بالليزر‬
and bipolar electrocoagulation (BEC) = Both are safe and effective ✅

- 689-Patient with rectal bleeding done technetium scan and Dx with angiodysplasia in left colon
TTT ?
laser ablation
keyword: rectal bleeding= laser ablation
ttt of angiodys. In gastric =partial or total gastrectomy
if in colon= endoscopic then Rt hemicoloctomy
definitive treatment for gastric angiodyplasia :
first line endoscopic ablation
sconce line=RT hemicolectomy
- 690-Old p'tient >50 ye'rs old. Alternating diarrhea and constipation. Physic'l is normal. Underwent
sigmoidoscopy, revealed multiple lesions in the dist'l sigmoid. Biopsy = 'adenocarcinoma'. Next
step in management? Colonoscopy
Keyword: colonoscopy to check metastasis
- 691-Case of perianal swelling , perrectal bleeding , investigated : it is 1 cm from anal verge
, biopsy: ((adenocarcinoma )) (rectal) Mx ? abdomen perineal resection ✅.

2019 ‫ من سنة‬11 – 10 – 9 ‫تجميعات قلوري تيم لشهر‬


- 692-20-year-old weight lifter developed abdominal hernia that is irreducible, tender. Imaging shows
air-fluid levels in small bowel and no free air in large bowel. Type? Incarcerated
Strangulated hernia: ischemic segment will cause symptoms such as erythema, leukocytosis and fever.
(Toxic patient)
Keyword: air fluid in small bowel =incarcerate hernia
Blood supply cut to bowel and air in large bowel = strangulated hernia
- 693-Most common cause of Small bowel obstruction ? adhesions✅ Large bowel ? neoplasm✅

- 694-4th day post OP (appendicitis) presented with diffuse abd pain, distension , vomiting and
sluggish bowel sounds Abd xray: Multiple air/fluid level
-ileus ✅ ‫ يكون معاها صوت االمعاء كسول او خاملة‬-adhesions and SBO ‫يزيد صوت االمعاء‬
(( sluggish‫ كسول او خامل‬bowel sound == ileus))
((Incresse bowel sound ==adhesions and SBO ))
Keyword: sluggish = obstruction in intestinal so we show multiple air fluid
- 695-Pt 6 yrs post abd operation has bloating and signs of obstruction cause? adhesions✅

- 696-case scenario of intussusception what is the gold standard investigation? barium enema best
intinal = us Gold stander =enema
A-reduction using enamas only on case of peritonitis✅
B-treated by immediate surgical intervention
C-recurrence is gigh after surgical treatment
D- Shock common complication surgical
‫منطقي الن الصبغة توضح كانك تشوفها بعينك انه حاصل تداخل باالمعاء‬
Keyword: intussuppcetion is rare in adult and gold stander in adult and children =enama
Initialed =US

- 697-Patient with symptoms of intussusception what is true about it: C/Recurrence is high after
surgical treatment✅✅
Keyword: intussusception =recurrent in pediatric
- 698-What type of cancer that should be screened for without symptoms : Colon ✅

- 699-28 male healthy, hx 2 month of abd pain with 2 bloody stool, proctoscope done with numerous
polyps covered the linings and multiple biopsy taken , no details in hs of family hx or sexual.. Dx:
A. familial polypoid✅ B. UC C. diverticulosis coli D. human papillomavirus polyp

- 700-Multiple polyps in colon and duodenal And bloody diarrhea? Familial Colon Cancer
Syndromes✅.
- 701-Long case with Thumbprint sign on abdominal x-ray ? ulcerative colitis or Ischemic colitis

- 702-Thumbprint sign in abdomen ? Pseudomembranous colitis AND ischemic colitis

- 703-Man with high grade dysplasia of the esophagus? Refer him for surgery

- 704-Anterior duodenal perforation tratment : Graham omental patch✅

- 705-A young healthy man medically free came complaining of indigestion problem otherwise he is
normal no vomiting no nausea no bloody diarrhea you gonna do ? functional dyspepsia

2019 ‫ من سنة‬11 – 10 – 9 ‫تجميعات قلوري تيم لشهر‬


❖ Trauma
- 706-Patient post RTA with warm peripherals Which type of shock? Neurogenic bez (warm
peripheral)

- 707-15year, fell and revived a supracondylar fracture above the elbow. Management?
If Brachial pulse present >> Reduction If Not present >> Surgical Exploration
Keyword:
- 708-loss of consciousness + loss of his conscious with dilated pupils ? Epidural hematom✅

- 709-What kind of shoulder dislocation.. difficulty in addiction and internal rotation? posterior
dislocation

- 710-abdominal trauma (by wooden stick) with painful RLQ pain and grey discharge, pain increased
by extending the leg. Next step? Give antibiotics ✅

- 711-Female had femoral fracture then after I think a week developed respiratory symptomes ?
Fat embolism syndrome ✅

- 712-Colles fracture in pedia what is the most important mangement ? closed reduction
‫من اسمها كوليس = كلوز‬Colles = Close

- 713-Picture of green stick and ask for treatment? A. close reduction with cast

- 714-pulse increased ? hypovolemic

- 715-Patient 72 years old have DM,,,For 3 month cant be stand from the chair What can happen to
him ? Fall ✅.

- 716-Most common site of aortic injury ? Proximal to left brachiocephalic

- 717-Blunt truma in chest or thoracic what to injured? left subclavian artery

- 718-Most common site of thoracic aortic blunt trauma? distal to left subclavian artery .
= subclavian = sub= ‫ صعب حادث باالورطى شريان مهم = صعب‬distal = ‫ربط = حادث في االورطى = بعيد الشر عليكم = بعيد‬
= left .‫هللا يبعدكم عنه = يبعدكم‬

- 719-after RTA pt hospital is away 40km what would you do ? call surgical oncall✅

A traumatic pt lost 25% of his blood..which of the following is the most suspected to be effected first
Pulse pressure ✅.
Keyword: ABC then surgical oncall

- 720-male involved in RTA in ER he is fully conscious and no abnormality in vital signs admitted
for observation and a FAST scan done ,.what do you expect to find? peritoneum free fluid✅

2019 ‫ من سنة‬11 – 10 – 9 ‫تجميعات قلوري تيم لشهر‬


- 721-Traum' patient. Intra abdominl hemorrhage and seen injury. Underwent laparotomy and
splenectomy. Which of the following will be low? Insulin✅.

- 722-best method to clear cervical trauma in ICU patient after motor vehicle accident? CT

- 723-after RTA pt hospital is away 40km what would you do? ABC .

- 724-pt with neck injury after RTA ? A-O2 mask. ✅ B-ETT. C-Coricothyrdoctomy
Remember ABCDE of primary survey.. O2 mask > intubation > chest tube

- 725-45 years old man had MVA presented with isolated head injury and coma for 5 days in ICU ,
the best way of feeding? Nasogastric tube feedings ✅.

- 726-Gastric cancer metastasizing to the liver? chemotherapy

- 727-Pt e gastric cancer & gross ascites , next step in management Abdominal paracentesis And
have flat foot? spring ligament

-728-pt fallen down from 3 meter height he felt severe pain and swelling at the lower Rt leg xray
showed commonuted fracture of the lower tibia what is the most accurate management? open
reduction, internal fixation and elevation
Keyword: comminuted fracture = open
- 729-Femoral fracture 30 d angulation child 4 years old ? Traction

- 730-patient with mid shaft femoral fracture, 30% anterior angulation ? Closed reduction with hip
spica cast

- 731-Most important thing to do before reduction of fracture ? Check vascular status(pulse)


-732-Head trauma post MVC at speed 130 km/hr. Tight seat belt. Patient is stable and alert. Most
appropriate next step in management? Abdominal ct
Keyword: most appropriate next step = CT

- 733-vascular problems in lungs class/group of pulmonary HTN ? 3


= ‫ = ربط = ثالثة = وريد وشريان ورئة‬vascular + lung

- 734-Battle sign? Basal skull fracture‫من اسمها‬

- 745-Ear secretion after trauma external ear was intact ? presented with ear bleeding, ruptured
eardrum = Basal skull fracture

- 736-RTA patient injures jugular foramen, what will happen?


. something happening to the vocal cord ✅✅
.‫ فالزم نلبس جاكيت‬vocal cord = ‫ جايكيت = نلبسه بالبرد = في البرد يسير لنا التهاب بال‬jugular = ‫ربط = جاكيوالر‬

- 737-Pt had trauma cause base skull fracture .. jugular foramen: ipsilateral vocal cords injury

2019 ‫ من سنة‬11 – 10 – 9 ‫تجميعات قلوري تيم لشهر‬


- 738-The most probable viral cause of repture of ear drum with abcess or pus ? RSV

- 739-Case of alcoholic patients with epigastric pain radiating to the back with x ray showing air
under diaphragm? duodenal perforation✅

- 740-Pt have RTA can ((shrug shoulder)) but can’t move elbow and lower limbs ? High spinal cord
injury

- 741-Man who was in MVA, hypotensive with slow heart rate and can't move legs or hands, what
caused his shock? high spinal cord injury =What is the cause of this hypotension? Upper spinal
ingury ✅
Keyword: spinal inj cause hypotension
- 742-Young male, post MVA Opens eyes spontaneously Responds to verbal Commands Shrugs
shoulders Shallow breathing Left chest wall contusion Cannot flex elbows or move lower limbs
Respiratory rate: ?BP: hypotensive Most likely diagnosis: High spinal injury

- 743-positive valgus test ? Medial collateral ligament sprain

- 744-Football player received a trauma to lateral side of his left knee, the patient now is complaining
of severe pain and swelling of the medial side of his left knee, positive valgus and (-) anterior
drawer and lachman, most likely diagnosis: Medial collateral ligament sprain

- 745-Player professional football with knee Injury in lateral side , medial knee swelling , lachman
and MCmurray test ( negative) ? Medial collateral ligament sprain

- 746-Pt came with instable knee. Ex femur come in front of tibia, which ligament injured ?
anterior cruciate ligament (ACL)
military with hx of prolonged standing c/o flat feet and pain in medial foot what tendon involved ?
spiral ligament also called calcanenoavicular
football player received trauma to lateral side of his left knee the pt now is complaining of sever
pain and swelling of the medial side of his left knee, positive valgus and lachman, most likely
diagnosis?
A- Medial meniscus tear
B- Lateral meniscus tear
C- Medial collateral ligament sprain✅
D- Lateral collateral ligament sprain

, ‫الحمدهلل الذي هدانا لهذا وماكنا لنهتدي لولا أن هدانا هللا‬


‫نسأل هللا أن يساهم هذا الملف في تحقيقنا للنجاح و للدرجات العالية‬
,
‫كل الشكر لقلوري تيم و دكتورة ألاء وجميع القائمين على ذلك من‬
‫تنسيق أو تصحيح‬
. ‫لا تنسوهم من خالص الدعاء بالتوفيق‬

2019 ‫ من سنة‬11 – 10 – 9 ‫تجميعات قلوري تيم لشهر‬


‫هذا وهللا ولي التوفيق ‪.‬‬

‫تجميعات قلوري تيم لشهر ‪ 11 – 10 – 9‬من سنة ‪2019‬‬

You might also like